r. m. m. - 27...tuzson, florin popovici, viorel gh. vodă, ovidiu pop, e. grosswald (usa), leroy f....

105
ROMANIAN MATHEMATICAL SOCIETY Mehedinți Branch WINTER EDITION 2020 R. M. M. - 27 ROMANIAN MATHEMATICAL MAGAZINE ISSN 2501-0099

Upload: others

Post on 01-Sep-2020

8 views

Category:

Documents


1 download

TRANSCRIPT

Page 1: R. M. M. - 27...Tuzson, Florin Popovici, Viorel Gh. Vodă, Ovidiu Pop, E. Grosswald (USA), Leroy F. Meyers (USA), Francisco Bellot (Spain). Mihály encze contributed to ’’Gamma’’with

ROMANIAN MATHEMATICAL SOCIETYMehedinți Branch

WINTER EDITION 2020

R. M. M. - 27 ROMANIAN MATHEMATICAL

MAGAZINE

ISSN 2501-0099

Page 2: R. M. M. - 27...Tuzson, Florin Popovici, Viorel Gh. Vodă, Ovidiu Pop, E. Grosswald (USA), Leroy F. Meyers (USA), Francisco Bellot (Spain). Mihály encze contributed to ’’Gamma’’with

Romanian Mathematical Society-Mehedinți Branch 2020

1 ROMANIAN MATHEMATICAL MAGAZINE NR. 27

ROMANIAN MATHEMATICAL

SOCIETY

Mehedinți Branch

ROMANIAN MATHEMATICAL MAGAZINE

R.M.M.

Nr.27-WINTER EDITION 2020

Page 3: R. M. M. - 27...Tuzson, Florin Popovici, Viorel Gh. Vodă, Ovidiu Pop, E. Grosswald (USA), Leroy F. Meyers (USA), Francisco Bellot (Spain). Mihály encze contributed to ’’Gamma’’with

Romanian Mathematical Society-Mehedinți Branch 2020

2 ROMANIAN MATHEMATICAL MAGAZINE NR. 27

ROMANIAN MATHEMATICAL

SOCIETY

Mehedinți Branch

DANIEL SITARU-ROMANIA EDITOR IN CHIEF

ROMANIAN MATHEMATICAL MAGAZINE-PAPER VARIANT ISSN 1584-4897

GHEORGHE CĂINICEANU-ROMANIA

EDITORIAL BOARD

D.M.BĂTINEȚU-GIURGIU-ROMANIA

CLAUDIA NĂNUȚI-ROMANIA

NECULAI STANCIU-ROMANIA

FLORICĂ ANASTASE-ROMANIA

DAN NĂNUȚI-ROMANIA

IULIANA TRAȘCĂ-ROMANIA

EMILIA RĂDUCAN-ROMANIA

DRAGA TĂTUCU MARIANA-ROMANIA

DANA PAPONIU-ROMANIA

GIMOIU IULIANA-ROMANIA

DAN NEDEIANU-ROMANIA

OVIDIU TICUȘI-ROMANIA

MARIA UNGUREANU-ROMANIA

DANIEL STRETCU-ROMANIA

ROMANIAN MATHEMATICAL MAGAZINE-INTERACTIVE JOURNAL ISSN 2501-0099 WWW.SSMRMH.RO

DANIEL WISNIEWSKI-USA

EDITORIAL BOARD VALMIR KRASNICI-KOSOVO

Page 4: R. M. M. - 27...Tuzson, Florin Popovici, Viorel Gh. Vodă, Ovidiu Pop, E. Grosswald (USA), Leroy F. Meyers (USA), Francisco Bellot (Spain). Mihály encze contributed to ’’Gamma’’with

Romanian Mathematical Society-Mehedinți Branch 2020

3 ROMANIAN MATHEMATICAL MAGAZINE NR. 27

CONTENT

HAPPY BIRTHDAY TO EDITOR-IN-CHIEF OF OCTOGON MATHEMATICAL MAGAZINE -

MIHÁLY BENCZE – 65 YEARS! – D.M. Bătinețu- Giurgiu, Daniel Sitaru and Neculai

Stanciu………………………………………………………………………................................................................4

ABOUT JI CHEN’S INEQUALITY – D.M. Bătinețu- Giurgiu...............................................................7

STRUCTURI ALGEBRICE (IV) – Vasile Buruiană………………………………………...……………................8

ABOUT SOME APPLICATIONS OF INTEGRAL CHEBYSHEV’S INEQUALITY – Florică

Anastase.............................................................................................. …………………...…….......12

ABOUT AN INEQUALITY IN TRIANGLE FROM RMM-2019 – Marin Chirciu…………………............18

A NEW DEMONSTRATION OF RAMUS-E.ROUCHE-BLUNDON DOUBLE INEQUALITY AND IT’S CONSEQUENCES – Marian Dincă………………...............................................................……………20 METRIC RELATIONSHIPS IN CHIRIȚĂ’S TRIANGLE – Daniel Sitaru, Claudia Nănuți…………..….25

SPECIAL PROBLEMS IN ROMANIAN MATHEMATICAL CONTESTS - Marian Ursărescu..........29

ABOUT CALCULUS OF SOME AMAZING LIMITS - Florentin Vișescu………………............................30

A SIMPLE PROOF FOR SIERPINSKY’S INEQUALITY - Daniel Sitaru …………..………………………..32

A DEFINITIVE WAY OF SOLVING MATHEMATICS PROBLEMS-Srinivasa Raghava……………….33

ABOUT PROBLEM IX.33-RMM 24,SPRING EDITION 2020-Marin Chirciu…………………………..36

PROPOSED PROBLEMS…………………………………….………………………………………………………………...39

INDEX OF PROPOSERS AND SOLVERS RMM-27 PAPER MAGAZINE.……………………………………104

Page 5: R. M. M. - 27...Tuzson, Florin Popovici, Viorel Gh. Vodă, Ovidiu Pop, E. Grosswald (USA), Leroy F. Meyers (USA), Francisco Bellot (Spain). Mihály encze contributed to ’’Gamma’’with

Romanian Mathematical Society-Mehedinți Branch 2020

4 ROMANIAN MATHEMATICAL MAGAZINE NR. 27

HAPPY BIRTHDAY TO EDITOR-IN-CHIEF OF OCTOGON MATHEMATICAL

MAGAZINE - MIHÁLY BENCZE – 65 YEARS !

By D.M. Bătineţu-Giurgiu, Daniel Sitaru and Neculai Stanciu

ABSTRACT. This paper is a dedicated to Mihály Bencze who is celebrating his 65 years.

Mihály Bencze

Mihály Bencze, born on 20 November 1954 in Săcele-Négyfalu, Braşov, Romania. He is

teacher of mathematics. He received Ph. D. in mathematics in 2010. He is Editor in Chief of

math journal Gamma (1978 - 1989) which becomes in 1993 Octogon Mathematical

Magazine; also he is editor in chief and member of many other scientific journals all over the

world. The first mathematical journal of the town Braşov (Brassó, Kronstadt) was founded by

Mihály Bencze in the year 1978, named ’’Gamma’’. Mihály Bencze was the editor-in-chief

and the members of the editorial board were the following: Constantin Ionescu-Ţiu, József

Sándor, Alexandru Lupaş, Florentin Smarandache, József Benkő, Alexandru Blaga, Zoltán

Tuzson, Florin Popovici, Viorel Gh. Vodă, Ovidiu Pop, E. Grosswald (USA), Leroy F. Meyers

(USA), Francisco Bellot (Spain). Mihály Bencze contributed to ’’Gamma’’with 9000 Proposed

Problems, 150 Open Question and 250 articles during 12 volumes between 1978-1989. In

April 1989 the ’’Gamma’’ journal was suppressed by the Romanian Communist party for

unclarified reasons, and till 1983 it had a latency period functioning as a prohibited journal.

In 1993, Mihály Bencze, laid down bases of the Octogon Mathematical Magazine written in

english and became the legal succesor of the former ’’Gamma’’ journal. During these years

Octogon Mathematical Magazine is published two times a year in April and October.

Page 6: R. M. M. - 27...Tuzson, Florin Popovici, Viorel Gh. Vodă, Ovidiu Pop, E. Grosswald (USA), Leroy F. Meyers (USA), Francisco Bellot (Spain). Mihály encze contributed to ’’Gamma’’with

Romanian Mathematical Society-Mehedinți Branch 2020

5 ROMANIAN MATHEMATICAL MAGAZINE NR. 27

Topics covered by Octogon Mathematical Magazine: general, inequalities, history and

biography, mathematical logic and foundations, combinatorics, number theory, general

algebraic systems, real functions, integrations, geometry, and others in all areas of pure and

applied mathematics. Aims and scope: Octogon Mathematical Magazine publishes high

quality original research papers and survey articles, proposed problems and open questions.

The Octogon Mathematical Magazine promote mathematical culture in the world of the

mathematical sciences and to facilitate the exchange of resources.

The publishing process in the Octogon Mathematical Magazine begins with the author's idea

for the work itself, continuing as a collaboration between the author and the editor as the

work is created. As such, editing is a practice that includes creative skills, human relations,

and a precise set of methods. The title of the top editor at many publications may be known

as the editor in chief, executive editor, or simply the editor. A frequent and esteemed

contributor to a magazine may acquire a title of editor at-large or contributing editor.

The top editor sometimes has the title executive editor or editor-in-chief. This person is

generally responsible for the content of the publication. An exception is large newspapers,

who usually have a separate editor for the editorials and opinion pages to separate news

reporting and editorial content. The editor-in-chief sets the publication standards for

performance, and motivates and develops the staff. The editor-in-chief is also responsible

for developing and maintaining the publication budget. In concert with the publisher and the

operating committee, the editor-in-chief is responsible for strategic and operational

planning. The editor-in-chief is effectively the head of this magazine and has considerable

influence on its content. One of the great strengths of the present editorial team of Octogon

is the ability of its members to cooperate on as a whole. The Octogon Mathematical

Magazine has thousand-pages of research papers which often becomes the next hundred-

pages articles and notes in a popular journals, and this keeps the volume of mathematics

manageable, what can be accessible to the students, graduate students or their teachers of a

future academic generation. One might think that such magazine are of interest only the

researchers and problem buffs, but this is not quite so. These articles, notes, proposed

problems and open questions – created by mathematicians, often with considerable

qualifications and experience, are used and adapted to become accessible to a wide range of

students and to foster some interesting investigations to highlight ideas, approaches and

Page 7: R. M. M. - 27...Tuzson, Florin Popovici, Viorel Gh. Vodă, Ovidiu Pop, E. Grosswald (USA), Leroy F. Meyers (USA), Francisco Bellot (Spain). Mihály encze contributed to ’’Gamma’’with

Romanian Mathematical Society-Mehedinți Branch 2020

6 ROMANIAN MATHEMATICAL MAGAZINE NR. 27

results, even if they cannot be directly imported into general classroom situations, they can

often be reworded. Even if the research cannot take into account all variables and cannot

answer definitively all of the questions, we might ask about mathematics, we can expect

research in mathematics to be helpful in many ways. The research it can: answer questions,

inform us, educate us, prompt new questions, create reflection and discussion, clarify

situations, help make educational decisions, challenge what we currently do as educators,

etc. Above all, despite educational research provides information that the community of

educators can use. Also we can use the research to shift from the ’’Yesterday’’ mind to the

’’Tomorrow’’ mind (see e.g. *3+).. The Octogon Mathematical Magazine is an inclusive

professional community of researchers and teachers who promote research in mathematics.

The Octogon Mathematical Magazine focuses on promoting research-based innovations

related to preparation and continued professional enhancement in mathematics. Target

audiences include teachers, researchers, higher education, faculty members, the secondary,

undergraduate and graduate levels.Mihály Bencze has published thousands of problems,

notes and articles in many scientific journal all over the world and also he has published

many books for Mathematical Contests. Mihály Bencze is: the organizer of Rubik’s contest

(1980) , the president of association Wildt József, the scientific leader of Fulgur publisher of

Braşov (since 1993), the founder of the International Hungarian mathematical Competition,

the founder of the Wildt József International Mathematical Competition. Also he is editor-in-

chief of 20 journals, editor to other 20 magazine and mentor for other 20 journals.

His creative work is amazing: has published over 1000 articles, 20 books and over 20.000

proposed problems and open questions in almost all the journals with columns of Problems

all over the world. The fact that Mihály Bencze, is a example for many pupils, students,

teachers and colleagues was proved in this paper and others to (e.g [6] , [7], [8], [9]). It has

been a great pleasure for us to have developed on ongoing friendship with such a modest

and such a refined and talented mathematician. Another important feature of his character

is the fact that he does not upset anyone, not ever judge people, is always happy and

respect all - and that means for him to be human.We are sure we speak on behalf of all

lovers of mathematics when we say:“Mihály Bencze may you live long, and continue to write

books, to submit articles, proposed problems and open questions to enjoy us”.

Mihály BenczeHappy birthday to our amazing Editor-In-Chief .

Page 8: R. M. M. - 27...Tuzson, Florin Popovici, Viorel Gh. Vodă, Ovidiu Pop, E. Grosswald (USA), Leroy F. Meyers (USA), Francisco Bellot (Spain). Mihály encze contributed to ’’Gamma’’with

Romanian Mathematical Society-Mehedinți Branch 2020

7 ROMANIAN MATHEMATICAL MAGAZINE NR. 27

ABOUT JI CHEN’S INEQUALITY

By D.M. Bătinețu-Giurgiu – Romania

Ji Chen’s inequality: If ( ), then:

( ) .

( )

( )

( ) /

(J.C)

and this inequality was published in Crux Mathematicorum magazine, vol. 20, Nr. 4, pp. 108, year 1995. This problem was also given to Iran’s Mathematical Olympiad in 1996.

Bellow we give a generalization of this inequality:

Theorem: Let be and , then:

( ( ) ( )

( ) ) . .

( )

( )

( )

/ / (*)

Proof: We have: ∑ ( ) ∑ ( ( )

) ⏞

( ) ∑ √ ⏟

( )

( )∑ (1)

and ∑

( )

.∑

( ) /

:∑

( )

;

( ) ∑ √ ⏟

(

( ) *

( )∑

( ) (2)

From (1) and (2) inequality (*) becomes:

: ∑( )

;: ∑

( ) ( )

;

( )( ):∑

;:∑

( )

;

( )( )

. Q.E.D.

Observation. If in inequality (*) we take we obtain Ji Chen inequality.

Page 9: R. M. M. - 27...Tuzson, Florin Popovici, Viorel Gh. Vodă, Ovidiu Pop, E. Grosswald (USA), Leroy F. Meyers (USA), Francisco Bellot (Spain). Mihály encze contributed to ’’Gamma’’with

Romanian Mathematical Society-Mehedinți Branch 2020

8 ROMANIAN MATHEMATICAL MAGAZINE NR. 27

STRUCTURI ALGEBRICE (IV)

By Vasile Buruiană – Romania

14. Fie inel integru comutativ. Atunci sunt echivalente:

a) este factorial

b) In nu există șiruri astfel ca și , și orice pereche de elemente din are cmmdc.

c) In nu există șiruri astfel ca și și orice element ireductibil din este prim.

Demonstrație: Pentru a) b) și a) c) este suficient și arătăm că nu există șiruri cu proprietatea anunțată. Dacă prin absurd ar exista astfel de șiruri notând ( ) numărul factorilor ireductibili în care se descompune ( ) ( ) ceea ce este absurd.

b) c) evident din Proprietatea 2.

c) a) Este sufficient să arătăm că orice element nenul și neinversabil din se descompune în produs finit de elemente ireductibile.

Dacă nu ar avea o descompunere finită un element atunci ar exista neasociat cu cu și nu ar avea descompunere finită cu și nu are descompunere finită continuând și luând , se construiește un șir cu și ceea ce contrazice ipoteza c).

15. Fie inel factorial și un sistem multiplicativ. Atunci este factorial (inelul de fracții pentru )

Demonstrație: Evident este integru. Fie apoi prim în , care nu este inversabil în

; el va fi prim și în . În adevăr fie

astfel ca

(unde

) sau . Cum (dacă este inversabil în )

sau

sau

. Cum orice

, nenul și neinversabil, se

descompune în produs finit de primi din , obținem afirmația.

16. Este posibil ca factorial dar să nu fie factorial (în particular este posibil ca să fie factorial dar să nu fie factorial).

În adevăr fie [ √ ] corpul său de fracții. este factorial ca orice corp, dar ,

am văzut, deja, nu este factorial.

17. Un ideal se numește prim dacă din relație sau . Evident în ideal din (inelul integru cu , neinversabil) este prim este prim.

Fie inel factorial. Atunci orice ideal prim nenul, minimal pentru relația de incluziune, este principal. În adevăr fie un ideal prim minimal, nenul astfel încât . Dacă

Page 10: R. M. M. - 27...Tuzson, Florin Popovici, Viorel Gh. Vodă, Ovidiu Pop, E. Grosswald (USA), Leroy F. Meyers (USA), Francisco Bellot (Spain). Mihály encze contributed to ’’Gamma’’with

Romanian Mathematical Society-Mehedinți Branch 2020

9 ROMANIAN MATHEMATICAL MAGAZINE NR. 27

∏ , căci este factorial, cu și cum este prim iar minimal

avem , q.e.d.

18. În inelul factorial , - singurele elemente ireductibile sunt , iar în , - singurele elemente ireductibile sunt cu sau cu și

În adevăr orice polinom din , - are rădăcinile complexe, deci se descompune în factori liniari. În cazul lui , - orice alt polinom în afara de cele enumerate este reductibil (căci ori are o rădăcină reală ori are rădăcini complex conjugate și se divide prin ( )( ).

Faptul că este ireductibil în , - dacă și numai dacă rezultă din aceea că dacă s-ar descompune în factori de gr. I ar avea rădăcini reale. Restul este evident.

19. Criteriul de ireductibilitate Eisenstein. Fie factorial cu corpul de fracții și , -,

și prim astfel încât

. Atunci este ireductibil.

Demonstrație. Putem presupune că este primitiv. Dacă ar fi reductibil în , -

și în , - ∑

∑ cu . Din și dar

sau dar nu pe amândouă. Presupunem că și . Fie minim astfel că

(există fiindcă deci nu toți ceficienții lui se divid la ). Dar

sau ceea ce este contradictoriu.

20. Dacă este morfism de inele integre, corpurile de fracții ale

, - , - ce extinde astfel ca ( ) factorial și , - astfel ca ( ) este ireductibil în , - și grad grad ( ) este ireductibil în , -

În adevăr, putem presupune primitiv. Dacă în , - el e reductibil în

, - ( ) ( ) ( ) și cum grad ( ) grad , grad ( ) grad .

( ) este reductibil, ceea ce este contradictoriu.

21. În , - polinomul este ireductibil. În adevăr în , - este ireductibil și ( )( ) ( )( ) ( ) și aplicând criteriul lui Eisenstein pentru și , rezultă afirmația.

La fel în , - este ireductibil. De asemenea în , - , este ireductibil pentru prim. În adevăr fie , - , - cu ( ) , morfism de

inele bijective ( ( )). Este suficient să arătăm că ( ) este

ireductibil. În adevăr

Page 11: R. M. M. - 27...Tuzson, Florin Popovici, Viorel Gh. Vodă, Ovidiu Pop, E. Grosswald (USA), Leroy F. Meyers (USA), Francisco Bellot (Spain). Mihály encze contributed to ’’Gamma’’with

Romanian Mathematical Society-Mehedinți Branch 2020

10 ROMANIAN MATHEMATICAL MAGAZINE NR. 27

( ) (

*

( )

și cum

și

folosind criteriul Eisenstein,

obținem afirmația.

In , - polinomul ( ) este reductibil.

În adevăr privit ca polinom în , -, - și luînd avem

( ) ( ) ( ) ( ) deci este reductibil.

22. In , - există polinoame ireductibile de orice grad. În adevăr și , folosind criteriul Eisenstein se vede că este ireductibil în , -.

23. Dacă este prim și atunci (Mica teoremă a lui Fermat)

Demonstrație. Prind inducție asupra lui . Pentru , afirmația este evidentă și presupunem că , voind să arătăm că ( ) ( ) . Dar

( ) ( ) ( ) ( ) ( ) ( )

( ) ( )

( ) (2)

Pentru * + avem

( ) ( )

și cum este prim,

folosind și ipoteza inductivă suma (2) se divide la

Dacă ( ) căci ( ) și ( ) deci .

Rezultă că în inelul claselor de resturi modulo , avem că polinomul

( )( ) . ( )/ de grad are rădăcini, deci

este identic nul, deci avem ( )( ) ( ( )) ( )

Facând obținem teorema Wilson: Dacă este număr prim atunci

( ) (mod ). Cum (mod) (mod )

(mod ), înmulțind ultimele relații și înlocuind în teorema Wilson

0.

/ 1

( )

(mod ) obținem .

/ ( )

(mod )

24. Elementele ireductibile în , - sunt:

a) elementele prime din astfel încât (mod )

Page 12: R. M. M. - 27...Tuzson, Florin Popovici, Viorel Gh. Vodă, Ovidiu Pop, E. Grosswald (USA), Leroy F. Meyers (USA), Francisco Bellot (Spain). Mihály encze contributed to ’’Gamma’’with

Romanian Mathematical Society-Mehedinți Branch 2020

11 ROMANIAN MATHEMATICAL MAGAZINE NR. 27

b) factori primi din , - ai elemntelor prime în și (mod )

c) și asociații săi.

Demonstrație: Mai întâi observăm că , -, prim ( ) , decompunerea în în factori ireductibili , prim, astfel ca ⁄ .

In plus nu există doi intregi primi neasociați astfel ca ⁄ ⁄ căci

( ) și , ceea ce arată că este contradictoriu. Această arată că elementele ireductibile din , - trebuie căutate printre divizorii din , - ai elementelor prime din . Fie ⁄ , unde (mod 4) ( ) ( ) deci ( ) și sunt posibile cazurile: ( ) (contradictoriu), ( ) (contradictoriu dacă ) deci ( ) . Luând . Însă * + deci nu putem avea căci (mod ) nu sunt decompozabile în , - a). Dacă ⁄ , unde (mod ) avem mai întâi, conform observației precedente,

că (.

/ *

(mod ) fiindcă

este par.

Deci există astfel ca (mod ), deci ( )( ). Cum

sau . Dacă

sau

, - deci

, ceea ce este absurd

nu poate fi inductibil în , - și factorii săi ireductibili sunt ireductibili în , - deci b)

În fine ( )( ) deci ( ) . Cum ( ) , dacă ar fi reductibil ar rezulta că ( ), deci ( ) ( ) (căci este neinversabil și deci ( ) ( ) și ( ) c).

25. (Reciproca teoremei Wilson ca un criteriu de recunoscut numere prime)

Dacă și ( ) (mod ) atunci este prim.

Demonstrație: Presupunem că este reductibil. Pentru relația (mod ) evident nu are loc, iar dacă avem cazurile:

I. * + ( ) și deci (mod ) ceea ce este absurd.

II. căci și * + ( ) deci iarăși (mod ) ceea ce este absurd.

Rezultă că pentru decompozabil nu este posibilă relația dată, deci este prim.

26. Metoda lui Kronecker de determinare a divizorilor unui polinom , -

și factorial, într-un număr finit de pași:

Mai întâi observăm dacă în , - avem ( ) ( ) ( ) deci

( ) ( ). Deci polinoamele ce divid pe au proprietatea că ( ) ( ) . In plus

unul din grad , grad

. Fie grad

.

Page 13: R. M. M. - 27...Tuzson, Florin Popovici, Viorel Gh. Vodă, Ovidiu Pop, E. Grosswald (USA), Leroy F. Meyers (USA), Francisco Bellot (Spain). Mihály encze contributed to ’’Gamma’’with

Romanian Mathematical Society-Mehedinți Branch 2020

12 ROMANIAN MATHEMATICAL MAGAZINE NR. 27

Apoi observăm că dacă , distincte reciproc și neasociate atunci

, există un unic polinom , - de grad astfel ca ( )

. În adevăr fie ( ) ( )( ) ( ) ( ) ( )

( ) ( )

unde ( ) este derivata lui adică ( ) ∑ ∏ ( )

Evident ( ) {

* +, grad . Luăm

∑ ( ) ( ) . Existența este demonstrată. Dacă ar fi alt

polinom cu ( ) are grad și rădăcini este polinom nul și deci rezultă unicitatea. Acum procedeul de determinare a divizorilor lui este următorul: Fie ( ) și ( ) în astfel ca . Pentru fiecare construim cu ( ) și verificăm dacă sau . Într-un număr finit de pași găsim toți divizorii sau stabilim că este ireductibil. Deși finite, totuși calculele sunt prohibitive.

ABOUT SOME APPLICATIONS OF INTEGRAL CHEBYSEV’S INEQUALITY

By Florică Anastase-Romania

In memory of my Professor LAURENȚIU MIRCEA PANAITOPOL

Theorem:( Chebysev’s Inequality)

For , - , continuous function with same increasing and , - , ) integrable function. Then:

:∫ ( )

;:∫ ( ) ( ) ( )

; :∫ ( ) ( )

;:∫ ( ) ( )

; ( )

In the case f and g different increasing:

:∫ ( )

;:∫ ( ) ( ) ( )

; :∫ ( ) ( )

;:∫ ( ) ( )

;

Demonstration:

If and are same increasing ( ) , -

( ) ( )( ( ) ( ))( ( ) ( )) , -

Page 14: R. M. M. - 27...Tuzson, Florin Popovici, Viorel Gh. Vodă, Ovidiu Pop, E. Grosswald (USA), Leroy F. Meyers (USA), Francisco Bellot (Spain). Mihály encze contributed to ’’Gamma’’with

Romanian Mathematical Society-Mehedinți Branch 2020

13 ROMANIAN MATHEMATICAL MAGAZINE NR. 27

( ) ( ) ( ) ( ) ( ) ( ) ( ) ( ) ( ) ( ) ( ) ( ) ( ) ( ) ( ) ( )

( )∫ ( ) ( ) ( ) ( ) ( )∫ ( ) ( )

( ) ( )∫ ( ) ( ) ( ) ( ) ( )∫ ( )

:∫ ( )

;:∫ ( ) ( ) ( )

; :∫ ( ) ( )

;:∫ ( ) ( )

;

:∫ ( ) ( )

;:∫ ( ) ( )

; :∫ ( ) ( ) ( )

;:∫ ( )

; ( )

For ( ) , -, we get:

) ( )∫ ( ) ( ) .∫ ( )

/ .∫ ( )

/

if and are same increasing;

) ( )∫ ( ) ( ) .∫ ( )

/ .∫ ( )

/

if and are different increasing;

Application 1:

If , - is continuous and increasing. Then:

∫ ( )

∫ ( )

Solution: If or is constant, then we have equalities. Let and variable on , -. Let function ( ) , -. Then and have same increasing on , - and from Chebyshev’s inequality (a):

:∫ ( )

;:∫

; ( )∫ ( )

∫ ( )

( )∫ ( )

∫ ( )

∫ ( )

Application 2: If , - continuous and derivable with ( ) , -. Then:

∫ ( )

( )

Solution:

Page 15: R. M. M. - 27...Tuzson, Florin Popovici, Viorel Gh. Vodă, Ovidiu Pop, E. Grosswald (USA), Leroy F. Meyers (USA), Francisco Bellot (Spain). Mihály encze contributed to ’’Gamma’’with

Romanian Mathematical Society-Mehedinți Branch 2020

14 ROMANIAN MATHEMATICAL MAGAZINE NR. 27

First: . ( )

/

( ) ( )

( )

is decreasing. Applying Chebyshev’s inequality, we

have:

∫ ( ) ∫ ( )

:∫ ( )

;

:∫

;

( )∫

( )

∫ ( )

Application 3:

If , - , ) are continuous and

are increasing, then:

∫(∫ ( )

∫ ( )

+ ∫ ( )

( )

Solution: From Chebyshev’s inequality:

∫ ( ) ∫ ( )

( ) ∫ ( )

( )

( ) ∫ ( )

∫ ( )

( ) ∫

( )

( )

∫ ( )

∫ ( )

( )

( ) , -

∫(∫ ( )

∫ ( )

+ ∫:

( )

( )

; ∫ ( )

( )( )

∫ ( )

( )

Application 4: If , - ( ) is decreasing, then:

∫ ( )

∫ ( )

∫ ( )

∫ ( )

Solution:

In Chebyshev’s inequality, let: ( ) ( )

( ) ( ) ( )

:∫ ( )

; :∫ ( )

( )

; :∫ ( )

; :∫ ( )

( )

;

:∫ ( )

; :∫ ( )

; :∫ ( )

; :∫ ( )

;

Application 5: If , - is twice derivable function, ( ) and ⏟

( ) then:

Page 16: R. M. M. - 27...Tuzson, Florin Popovici, Viorel Gh. Vodă, Ovidiu Pop, E. Grosswald (USA), Leroy F. Meyers (USA), Francisco Bellot (Spain). Mihály encze contributed to ’’Gamma’’with

Romanian Mathematical Society-Mehedinți Branch 2020

15 ROMANIAN MATHEMATICAL MAGAZINE NR. 27

( ) ∫ ( )

Solution: Applying Chebyshev’s inequality function ( ) ( )

∫ ( )

( )

:∫ ( )

;

( ) ∫ ( )

( ) ∫ ( )

:∫ ( )

;

|∫ ( )

|

Application 6: Prove:

.

/

Solution:

Let 0

1 ( )

( )

( )

decreasing and ( ) increasing for 0

1.

∫ √

( )( )

.

/

.

/

Application 7: Prove:

( )( )

Solution:

( )( ) ∫(

* ∫

Application 8: If , - , - is twice derivable, ( ) , - are increasing and convexe on , -, then:

Page 17: R. M. M. - 27...Tuzson, Florin Popovici, Viorel Gh. Vodă, Ovidiu Pop, E. Grosswald (USA), Leroy F. Meyers (USA), Francisco Bellot (Spain). Mihály encze contributed to ’’Gamma’’with

Romanian Mathematical Society-Mehedinți Branch 2020

16 ROMANIAN MATHEMATICAL MAGAZINE NR. 27

) ( ( )) ( ( )) ( ( ) ( )) 4 ( ) ( )

5

) (

*

(

*

Solution:

a) ( ) , - increasing on , - and convexe ⏞

( ( )) ( ( )) ∫( ) ( )

∫ ( ( )) ( )

∫ ( ( )) ∫ ( )

( ) ( )

∫ ( ( ))

( ( ) ( )) :

∫ ( )

; ( ( ) ( )) 4 ( ) ( )

5

b) Let 0

1 0

1 ( ) ( )

( )

( )

( )

4 (

*5 ( ( )) ( (

* ( )* 4

( ) ( )

5

(

*

(

*

Application 9: If

, then:

∫:

∫ .

/

;

∫ .

/

Solution:

First: ∫

( )

Let be the functions: 0

1 ( )

( ) ∫ ( ) ( ) .

/

and

( ) ∫ ( )

How: ( ) ( )

( ) ( )

( )( )

0

1 is convexe , - and such that:

(( ) ) ( ) ( ) ( ) for

( )

( )

( )

is increasing (analogous

( )

is increasing

( ) decreasing)

Applying Chebyshev’s inequality, we get:

Page 18: R. M. M. - 27...Tuzson, Florin Popovici, Viorel Gh. Vodă, Ovidiu Pop, E. Grosswald (USA), Leroy F. Meyers (USA), Francisco Bellot (Spain). Mihály encze contributed to ’’Gamma’’with

Romanian Mathematical Society-Mehedinți Branch 2020

17 ROMANIAN MATHEMATICAL MAGAZINE NR. 27

∫ ( )

( ) ∫

( )

( )

( )

( ) ( )

( )

( )

.

/

( )

∫ .

/

Application 10: If and then:

∫ ( )

( √ ) ( √ )

Solution:

∫ ( )

√ ∫

√ ( )

From Chebyshev’s inequality, let: ( )

integrable and ( ) √

( ) ( ) are increasing

(

)

(

∫ ( )

)

(

∫√

)

(

∫ ( )

)

( )

.

/

( )

∫√

( √ )

( )

∫ ( )

∫ .

/

(.

/

*

.

/

.

/

.

/

.

/

.

/

( )

∫ ( )

( √ )

( )

From (i),(ii),(iii),(iv), we get:

( )

( √ ) ( √ )

∫ ( )

( √ ) ( √ )

Page 19: R. M. M. - 27...Tuzson, Florin Popovici, Viorel Gh. Vodă, Ovidiu Pop, E. Grosswald (USA), Leroy F. Meyers (USA), Francisco Bellot (Spain). Mihály encze contributed to ’’Gamma’’with

Romanian Mathematical Society-Mehedinți Branch 2020

18 ROMANIAN MATHEMATICAL MAGAZINE NR. 27

Application 11: If , then:

4

√ 5

Solution:

Let: 0

1 ( )

( )

derivable with

( ) ( )

( ) ( )

( ) is increasing and decreasing

( ) ( )

( )

( )

( )

( )

( )

Let

.

( ) /

( )

( )

From (ii), (iii) we get:

|

√ . So:

4

√ 5

Refferences: Romanian Mathematical Magazine-Interactive Journal-www.ssmrmh.ro

ABOUT AN INEQUALITY IN TRIANGLE FROM RMM-2019

By Marin Chirciu – Romania

1) In – centroid and – circumradii respectively.

Prove that:

Proposed by Marian Ursărescu – Romania

Solution: We prove the following lemma:

Lemma:

2) In – centroid and – circumradii respectively.

Prove that:

Page 20: R. M. M. - 27...Tuzson, Florin Popovici, Viorel Gh. Vodă, Ovidiu Pop, E. Grosswald (USA), Leroy F. Meyers (USA), Francisco Bellot (Spain). Mihály encze contributed to ’’Gamma’’with

Romanian Mathematical Society-Mehedinți Branch 2020

19 ROMANIAN MATHEMATICAL MAGAZINE NR. 27

Expressing the area of in two ways, we obtain:

, -

and , -

, wherefrom

It follows

and from here

We have:

Let’s get back to the main problem:

Using the Lemma and inequality we obtain:

∑ ∑

( )

, so, ∑

Using the known identity in triangle

.

In order to prove the inequality from the enunciation, it suffices to prove that:

, which follows from Gerretsen’s inequality:

. It suffices to prove that: (Euler’s

inequality). Equality holds if and only if the triangle is equilateral.

Remark: Let’s find an inequality having an opposite sense:

3) In – centroid and – circumradii respectively.

Prove that:

.

/

Proposed by Marin Chirciu – Romania

Solution: Using the Lemma and Tereshin’s inequality

, we obtain:

∑( )( )

( ) ( )

, which follows from

∑( ) ( ) ( ) ( ) , so

( ) ( )

Using the known identity in triangle:

.

In order to prove the inequality from enunciation it suffices to prove that:

Page 21: R. M. M. - 27...Tuzson, Florin Popovici, Viorel Gh. Vodă, Ovidiu Pop, E. Grosswald (USA), Leroy F. Meyers (USA), Francisco Bellot (Spain). Mihály encze contributed to ’’Gamma’’with

Romanian Mathematical Society-Mehedinți Branch 2020

20 ROMANIAN MATHEMATICAL MAGAZINE NR. 27

( ) ( )

( ) ( ) ( ), which follows from Gerretsen’s

inequality ( )

. It suffices to prove that:

( )

( ( ) ) ( ) ( )

( )( ) , true from Euler’s inequality

. Equality holds if and only if the triangle is equilateral.

Remark: The double inequality can be written:

4) In – centroid and – circumradii respectively.

Prove that:

.

/

Solution

See inequalities 1) and 3). Equality holds if and only if the triangle is equilateral.

Refferences:

Romanian Mathematical Magazine-Interactive Journal-www.ssmrmh.ro

A NEW DEMONSTRATION OF RAMUS-E.ROUCHE-BLUNDON DOUBLE

INEQUALITY AND IT’S CONSEQUENCES

By Marian Dincă – Romania

Page 22: R. M. M. - 27...Tuzson, Florin Popovici, Viorel Gh. Vodă, Ovidiu Pop, E. Grosswald (USA), Leroy F. Meyers (USA), Francisco Bellot (Spain). Mihály encze contributed to ’’Gamma’’with

Romanian Mathematical Society-Mehedinți Branch 2020

21 ROMANIAN MATHEMATICAL MAGAZINE NR. 27

THEOREM: Let be a triangle that has its circumcircle ( ) and the incircle ( ) Then, it exists two isosceles triangles inscribed to the circle ( ) and circumscribed to the circle ( ).

Proof: Without using Poncelet theorem we use Carnot identity:

We will prove that there exists two angles respectively such that:

.

/ (

*

Such that: . Elementary proof:

.

/ .

/ .

/

or: .

/ .

/

.

/

√ ( )

√ ( )

√ .

/

So:

, it follows:

, let be

respectively:

, let be

Let be: , .

/

but: .

/ .

/ .

/

It follows: .

/ .

/

So: .

/ .

/. The function:

( ) .

/ 0

1, ( ) .

/ , so, increasing

( ) ( ), implies: . Also: .

/ .

/

.

/

But: .

/ .

/

So: .

/ .

/.

The function: ( ) .

/ 0

1

( ) .

/ , decreasing, so:

In the end there are two isoscel triangles that satisfy Carnot relationship:

.

/ (

*

And: . Let be an isoscelles triangle with:

.

/ .

/

.

/

we prove that: (i)

( ) (

*

Or:

.

/ (ii)

Page 23: R. M. M. - 27...Tuzson, Florin Popovici, Viorel Gh. Vodă, Ovidiu Pop, E. Grosswald (USA), Leroy F. Meyers (USA), Francisco Bellot (Spain). Mihály encze contributed to ’’Gamma’’with

Romanian Mathematical Society-Mehedinți Branch 2020

22 ROMANIAN MATHEMATICAL MAGAZINE NR. 27

.

/ , it follows: .

/ , it follows:

.

/ .

/. So:

, it follows:

, then it exists and , - such that:

.

/ ( ) (1)

.

/ ( ) (1’)

, implies: .

/ ( ) .

/ ( )

Or: ( ) .

/ ( ) , or ( ) .

/

It follows:

( ) .

/ ( )

0 .

/ 1

Or: .

/ .

/ .

/

Or: .

/ .

/ , it follows:

Inequality (ii) is equivalent with:

: .

/ ( )

; :

.

/ ( )

;

: .

/ ( )

.

/ ( )

;

4.

/

( )5 4.

/

( )5

(.

/ (

* (

*+

4.

/

( )5 4.

/

( )5

(.

/ (

* (

*+

[ (

*

] [ (

*

] [(

* (

* ]

Let be , - , -

( ) [ (

*

] [ (

*

] [(

* (

* ]

is a convex function defined on a convex set: , - , - 2( )

3

So the pentagon , where: ( ) ( ) .

/ .

/ ( ). Will attain its

maximum in one of these peaks. Any convex function defined on a convex set will attain its maximum in its extreme points.

Page 24: R. M. M. - 27...Tuzson, Florin Popovici, Viorel Gh. Vodă, Ovidiu Pop, E. Grosswald (USA), Leroy F. Meyers (USA), Francisco Bellot (Spain). Mihály encze contributed to ’’Gamma’’with

Romanian Mathematical Society-Mehedinți Branch 2020

23 ROMANIAN MATHEMATICAL MAGAZINE NR. 27

( ) .

/

(

* .

/

(

*

.

/

(

* .

/ (

(

*

* (

*

.

/ .

/

.

/, or:

(

* .

/

, it follows:

( )

.

/

, -

(

* .

( )

/ .

/ ( )

So: .

/ .

/ ( )

We obtain:

.

/ .

/

.

/ ( )

.

/

( ) .

/ ( ( )) .

/

6

4.

/57

[

] (

*

But: ( ) ( ) and .

/ .

/ being symmetric.

.

/ , it follows: .

/

We prove the other inequality:

, or so:

, it follows:

.

/ we obtain:

Then it exists respectively , -, such that:

( ) (

* ( ) (

*

Page 25: R. M. M. - 27...Tuzson, Florin Popovici, Viorel Gh. Vodă, Ovidiu Pop, E. Grosswald (USA), Leroy F. Meyers (USA), Francisco Bellot (Spain). Mihály encze contributed to ’’Gamma’’with

Romanian Mathematical Society-Mehedinți Branch 2020

24 ROMANIAN MATHEMATICAL MAGAZINE NR. 27

( ) ( ) (

* ( ) (

*

It follows: , because

( ) (

* ( ) (

*

( ) ( ) (

*

[ (

*]

It follows: .

/ .

/ .

/

Or: .

/ .

/ . It follows:

( ( ) (

*+ ( ( ) (

*+

[ ( ) (

* ( ) (

*]

[ (

*

] [ (

*

]

[( ) (

* ] ( )

, - , -

is concave defined on the convex set , - , - 2( )

3

It will attain its maximum in the points .

/ ( ) ( ) ( ) .

/

Being simetric it remains to verify .

/ ( ) ( )

(

* (

(

*

* (

* (

(

* *

(

* (

*

(

*

Or:

.

/ .

/. Using the same technique:

Or Steffensen in discreete form

(

* (

* (

*

(

*

Ramus-E, Rouche, Blundon for any acute angled triangle: If triangle is acute-angled, that has the circumcircle triangle ( ), and the

inscribed circle ( ) and 0

1. Then it exists two acute-angled triangles that

has the same respectively and it exists the double inequality: where are semiperimeters of the those triangles.

Proof: In the first part of the article we’ve proved that: In the situation that the triangle is acute-angled, we can replace * + with

2

3 the triangle that has the angles having the measure:

, is acute

angled and:

, and

Page 26: R. M. M. - 27...Tuzson, Florin Popovici, Viorel Gh. Vodă, Ovidiu Pop, E. Grosswald (USA), Leroy F. Meyers (USA), Francisco Bellot (Spain). Mihály encze contributed to ’’Gamma’’with

Romanian Mathematical Society-Mehedinți Branch 2020

25 ROMANIAN MATHEMATICAL MAGAZINE NR. 27

In the same way like in the first part it will follow: .

/ (

* .

/

.

/ .

/ .

/ 4

5. So:

Bibliography: [1] R. Tyrrel Rockafellar: Convex Analysis, Theta Publishing House, 2002. [2] Arthur Engel: Mathematical problems, Solving strategies, GIL Publishing House, 2006. [3] Shan – He Wu, Yu-Ming Chu: Geometric interpretation of Blundon’s inequality and Ciamberlini’s inequality, Journal of inequalities and Applications, December 2014 Bellow we illustrate the proof: O246. Let be a point inside or on the boundry of a convex polygon . Prove that the maximum value of ( ) ∑

is achieved when is a vertex

Proposed by Cosmin Pohoata, Princeton University, USA

Solution by G.R A. 20 Problems Solving Group, Roma, Italy Let be the set of points inside or on the boundry of the convexe polygon. We first note that the map ( ) is convex: if with then, for any , we have that and ( ) ∑ , ( ) ( )-

( ) ( )

Moreover, for any point in is a convex combination of the vertices that is, there exists such that ∑

and ∑

Hence, by the covexity of ,

( ) (∑

+ ∑

( ) ∑

( )

( )

which means that the maximum value of ( ) is attained when is a vertex of .

METRIC RELATIONSHIPS IN CHIRIȚĂ’S TRIANGLE

By Daniel Sitaru, Claudia Nănuți-Romania

Abstract: In the romanian mathematician Marcel Chiriță proved that for any

√ √ √ can be sides in triangle.

In this paper we prove some metric relationships in this kind of triangle.

Proposition 1. Chiriță’s triangle is obtuse.

Proof: Let √ √ √

√( )( )

Page 27: R. M. M. - 27...Tuzson, Florin Popovici, Viorel Gh. Vodă, Ovidiu Pop, E. Grosswald (USA), Leroy F. Meyers (USA), Francisco Bellot (Spain). Mihály encze contributed to ’’Gamma’’with

Romanian Mathematical Society-Mehedinți Branch 2020

26 ROMANIAN MATHEMATICAL MAGAZINE NR. 27

√( )

( )

( )

Proposition 2. Area of Chiriță’s triangle is constant for any real value of .

Proof:

4 ( )

√ 5

( )

( )

( )

, -

√ √

Proposition 3. A median of Chiriță’s triangle is constant for any real value of .

Proof:

( )

( )

( )

( )

Proposition 4. If then:

√ √ √ √ √

Proof.

( )

( )

( )

( )

( )

( )

( ) √ (1)

( )

( )

( )

Page 28: R. M. M. - 27...Tuzson, Florin Popovici, Viorel Gh. Vodă, Ovidiu Pop, E. Grosswald (USA), Leroy F. Meyers (USA), Francisco Bellot (Spain). Mihály encze contributed to ’’Gamma’’with

Romanian Mathematical Society-Mehedinți Branch 2020

27 ROMANIAN MATHEMATICAL MAGAZINE NR. 27

( )

( )

( )

( ) √ (2)

It’s known that:

√ (3)

√ (4)

By adding (3); (4): √ √

√ √ √

Proposition 5. If then:

√ √ √ √( )( )

Proof.

√( )( )

By Mitrinovic’s inequality:

( )( )

√( )( )

√ √ √ √( )( )

Proposition 6. If then:

.√ √ /

.√ √ /

.√ √ /

Proof 1 (Soumava Chakraborty-Kolkata-India)

.√ √ /

.√ √ /

.√ √ /

⏞( )

(1) ( )

√( )( ) √( )( )

Page 29: R. M. M. - 27...Tuzson, Florin Popovici, Viorel Gh. Vodă, Ovidiu Pop, E. Grosswald (USA), Leroy F. Meyers (USA), Francisco Bellot (Spain). Mihály encze contributed to ’’Gamma’’with

Romanian Mathematical Society-Mehedinți Branch 2020

28 ROMANIAN MATHEMATICAL MAGAZINE NR. 27

√( )( ) (squaring (1))

( ) *( )( ) ( )( ) ( )( )+

8( )√( )( ) ( )√( )( )

( )√( )( )9

(squaring again) ( ) ( )

{

( )√( )( )

( )√( )( )

( )√( )( )

} (2)

.

/

and .

/

,

of (2) {( ) .

/ (√ ) ( ) .

/ .

/

( )(√ ) .√

/

}

( )

( )

of (2) (2) is true (Proved)

Proof 2.

√ √ √

In with sides :

( )

By Hadwiger – Finsler’s inequality: ∑( ) √

∑( ) √ √

∑( )

.√ √ /

.√ √ /

.√ √ /

Refferences:

1.Romanian Mathematical Magazine-Interactive Journal-www.ssmrmh.ro

2.Mihaly Bencze,Daniel Sitaru,Marian Ursărescu:”Olympic Mathematical Energy”-Studis-

Publishing House-Iași-2018

Page 30: R. M. M. - 27...Tuzson, Florin Popovici, Viorel Gh. Vodă, Ovidiu Pop, E. Grosswald (USA), Leroy F. Meyers (USA), Francisco Bellot (Spain). Mihály encze contributed to ’’Gamma’’with

Romanian Mathematical Society-Mehedinți Branch 2020

29 ROMANIAN MATHEMATICAL MAGAZINE NR. 27

3.Daniel Sitaru, George Apostolopoulos:”The Olympic Mathematical Marathon”-Publishing

House- Cartea Românească-Pitești-2018

4. Mihaly Bencze,Daniel Sitaru:”Quantum Mathematical Power” -Publishing House Studis,

Iasi-2018

5.Daniel Sitaru,Marian Ursărescu:”Calculus Marathon”-Studis-Publishing House-Iasi-2018

6.Daniel Sitaru, Mihaly Bencze:”699 Olympic Mathematical Challenges”-Publishing House

Studis, Iasi-2017

7.Daniel Sitaru,Marian Ursărescu:”Ice Math-Contests Problems”-Studis-Publishing House-

Iasi-2019

SPECIAL PROBLEMS IN ROMANIAN MATHEMATICAL CONTESTS

By Marian Ursărescu-Romania

ABSTRACT: In this article are presented some problems with matrices and determinants used as subjects in romanian mathematical contests. Lemma. If ( ) are such that: a) , b) Matrix is invertible. c) , , then ( ) Demonstration: Second degree equation with roots is: ( ) ( ) are roots of equation (1)

We have: ( )( ) ⏞( )

( ) ( ) ⏞( )

( ) ( ) ( )( ) ( )( ) ( )( )

,( )( )- ,( )( )-

( ) ( ) [( )( ) ] ( ) ( )

( ) If ( ) then ( ) and ( ) . Applications: 1) If ( ) such that and is invertible, then . Demonstration: By lemma, we have with and

By lemma ( ) but

2) If ( ) with

( ) and is invertible,

then RMO

Demonstration:In condition of lemma, we have

and such that

Page 31: R. M. M. - 27...Tuzson, Florin Popovici, Viorel Gh. Vodă, Ovidiu Pop, E. Grosswald (USA), Leroy F. Meyers (USA), Francisco Bellot (Spain). Mihály encze contributed to ’’Gamma’’with

Romanian Mathematical Society-Mehedinți Branch 2020

30 ROMANIAN MATHEMATICAL MAGAZINE NR. 27

.

By lemma: .

/

.

/

.

/

3) If ( ) such that ( ) √ √ and is invertible, then

RMO

Demonstration: In conditions of lemma, we have: √ √

and

√ √

Second degree equation have roots: √ √ (√ )

By lemma ( ) ( ) . But

.

/

Applications:

1) If ( ) such that ( √ )( ) and is invertible,

then . GMB

2) If ( ) such that . √ √ / ( ) (√ )( ) and

is invertible, then RMM

3) If ( ) such that √( √ √ ( ) √ √ √ ( ) and

is invertible, then . RMM

4) If ( ) such that √ √ ( ) √ √ ( ) and is invertible, then

RMM Refferences Romanian Mathematical Magazine-Interactive Mathematical Journal-www.ssmrmh.ro

ABOUT CALCULUS OF SOME AMAZING LIMITS

By Florentin Vișescu-Romania

Lemma: Let be and , - , ) a continuous function, then:

∏4

(

*5

Proof:

Page 32: R. M. M. - 27...Tuzson, Florin Popovici, Viorel Gh. Vodă, Ovidiu Pop, E. Grosswald (USA), Leroy F. Meyers (USA), Francisco Bellot (Spain). Mihály encze contributed to ’’Gamma’’with

Romanian Mathematical Society-Mehedinți Branch 2020

31 ROMANIAN MATHEMATICAL MAGAZINE NR. 27

∏4

(

*5

∏ (

.

/*

∑ (

.

/*

We prove that

( ) , ).

Consider , ) ( ) ( )

( )

, )

on , ) ( ) ( ) , ) or ( ) , )

consider , ) ( ) ( )

( )

( ) on , ) ( ) ( ) , ) or ( )

, ). So,

( ) , ).We choose

(

*

(

*

4

(

*5

4

(

*5

(

*

(

*

∑4

(

*5

∑ 4

(

*5

(

*

(

* ∫ ( )

(

*

(

*

∫ ( )

Then ∑ (

.

/*

∫ ( )

and so,

∏4

(

*5

∫ ( )

1) ∏ .

.

/

/

2) ∏ 4 √

5

. √( )

/

3) ∏ ( √

*

Page 33: R. M. M. - 27...Tuzson, Florin Popovici, Viorel Gh. Vodă, Ovidiu Pop, E. Grosswald (USA), Leroy F. Meyers (USA), Francisco Bellot (Spain). Mihály encze contributed to ’’Gamma’’with

Romanian Mathematical Society-Mehedinți Branch 2020

32 ROMANIAN MATHEMATICAL MAGAZINE NR. 27

4) ∑ .

/

(√ )

5) ∏ .

/

A SIMPLE PROOF FOR SIERPINSKY’S INEQUALITY

By Daniel Sitaru – Romania

Abstract: In this paper we give a proof by mathematical induction for Sierpinsky’s

inequality.

SIERPINSKY’S INEQUALITY:

For

(√ )

For

:

;

( )

( )

( ) ( )

( ) ( )

( )

(

*

Page 34: R. M. M. - 27...Tuzson, Florin Popovici, Viorel Gh. Vodă, Ovidiu Pop, E. Grosswald (USA), Leroy F. Meyers (USA), Francisco Bellot (Spain). Mihály encze contributed to ’’Gamma’’with

Romanian Mathematical Society-Mehedinți Branch 2020

33 ROMANIAN MATHEMATICAL MAGAZINE NR. 27

( )

( ) ( )

( ) ( )

( )

( ) suppose true

( ) to prove

( )

( ) ( )

A DEFINITIVE WAY OF SOLVING MATHEMATICS PROBLEMS K. SRINIVASA RAGHAVA

[email protected]

ALL INDIA RAMANUJAN MATHS CLUB

The difficulty in solving mathematics problems was the most common answer in the survey I conducted, when I asked if you have problems with mathematics. For this reason I will try to give some light to this need that we have raised.

STRATEGIES TO SOLVE MATHEMATICS PROBLEMS You get good marks in the exercises and you get stuck in the problems, right? It is normal! It also happened to the most for a while. Clear! If you barely spend time teaching us problem solving in class! Solving problems is the heart of mathematics, and I believe that more emphasis should be placed on teaching methodologies for resolution. What satisfaction when you succeed! Eureka! You may not believe it much, but you can increase your motivation and your ideas by facing appropriate problems.

EXERCISES AND PROBLEMS They are very different things. You are used to doing more exercises, so you get much better. An exercise is usually a mere practical application of a formula or calculation. At a glance you know what you have to do. You already know the way and you just have to apply it. A problem is necessary to read it carefully to understand it well. Here you ignore the road! Walker there is no way; you make your way when you walk. You have to organize and relate your ideas and have a positive and creative attitude. Problems are less defined and more open issues than exercises. Do not focus on memorizing theory and doing repetitive exercises. It is much more productive and fun to spend time solving problems.

Page 35: R. M. M. - 27...Tuzson, Florin Popovici, Viorel Gh. Vodă, Ovidiu Pop, E. Grosswald (USA), Leroy F. Meyers (USA), Francisco Bellot (Spain). Mihály encze contributed to ’’Gamma’’with

Romanian Mathematical Society-Mehedinți Branch 2020

34 ROMANIAN MATHEMATICAL MAGAZINE NR. 27

A GOOD FRIEND I think it's about time you start considering problems as friends that offer you the opportunity to progress in an active way in mathematics and to check what you know. Even sometimes, you can enjoy thinking and have "a happy idea" to come up with a simple and elegant solution. Solving a problem requires time and energy. It implies some circumstances like initial frustration, willingness to resolve it, perseverance But solving problems will increase your confidence with mathematics. You will learn more. Worth the effort!

STEPS TO SOLVE MATHEMATICAL PROBLEMS

It is true that solving mathematics problems is a complex activity. It may seem like a messy activity, but it is essential to organize your ideas and be schematic. These tips will help you think better:

You must have a positive attitude. Curiosity and desire to learn. Taste for the challenge

Trust your possibilities. We are what we think. Act calmly, without fear.

Be patient. Do not give up at the slightest difficulty. If you get stuck, think about a new approach to

the problem.

Concentrate. Solving problems is a complex activity and requires attention.

Don't look for short term success. Reaching the solution is a slow process, but when you

notice the progress you will feel great satisfaction.

There are really no definite strategies to solve mathematics problems that will ensure success. But if we can point out some general steps.

1. UNDERSTAND THE PROBLEM Read the sentence calmly. Several times, until you understand it well. Do not miss any interesting facts. What does it consist of? What do you know, what are you asked for? What are the conditions…? This is necessary to face the problem with guarantees of success.

2. PREPARE AN ACTION PLAN When you understand the problem, it is time to choose a strategy to solve it. There are many strategies! I indicate some at the end of the article. It is good that you know them and practice them to improve your ability to solve problems.

3. CARRY ON YOUR PLAN Once you have chosen a strategy, work it decisively and do not abandon it to the first difficulty.

Page 36: R. M. M. - 27...Tuzson, Florin Popovici, Viorel Gh. Vodă, Ovidiu Pop, E. Grosswald (USA), Leroy F. Meyers (USA), Francisco Bellot (Spain). Mihály encze contributed to ’’Gamma’’with

Romanian Mathematical Society-Mehedinți Branch 2020

35 ROMANIAN MATHEMATICAL MAGAZINE NR. 27

It is possible that things get complicated and you were wrong to choose a strategy. Another test! There are usually several ways to arrive at the solution and we cannot always succeed with the most appropriate one at the first attempt. He left? Are you sure? Review the result and verify that you have reached the solution. Many times we believe we have solved a problem and then it is not so.

4. REFLECT ON THE ENTIRE PROCESS «Every problem I solved became a rule that later helped me solve other problems. » Descartes Have you solved the problem?!! Congratulations!! Have you had a good time entertaining, trying hard, and ended up not solving it? Congratulations too! You learn much more from the problems worked with interest and determination, and not resolved, than from those that are solved almost at first sight. How did you solve it? This stage is very helpful and is often forgotten.

Examine well the path you have followed. How did you get to the solution? Or, why haven't you

reached the solution? What mistakes and successes have you had? What made you intuit that it was

going to go well?

See if you can do it in a simpler way.

Reflect a little on your thinking process and draw consequences for the future. Each person has a

different way of thinking.

How is your thought? Visual or analytical? Everything can be improved. With practice you can go from having a single rigid idea to having several related and original ideas.

STRATEGIES TO SOLVE MATHEMATICS PROBLEMS

Look for similarities with other problems. Does it remind you of any similar situation?

Reduce the complicated by something simpler. Divide and you will win!

Consider particular cases. Keep track! Use very small numbers

Make a drawing or scheme. A picture is worth a thousand words. It incorporates only the important.

Study all possible cases. Can you discard any?

Choose a good notation. You will simplify the problem a lot

Trial and error. If it doesn't work, take another path.

Work backwards. Imagine that the problem is solved and that you are a crab. It is possible that you

can build the solution.

Take advantage of symmetry. You may be able to take advantage of regularities or symmetries.

I hope that from now on solving mathematics problems become a pleasant distraction for you.

Who knows, one day you might say, "I like problems!" Or not?

Page 37: R. M. M. - 27...Tuzson, Florin Popovici, Viorel Gh. Vodă, Ovidiu Pop, E. Grosswald (USA), Leroy F. Meyers (USA), Francisco Bellot (Spain). Mihály encze contributed to ’’Gamma’’with

Romanian Mathematical Society-Mehedinți Branch 2020

36 ROMANIAN MATHEMATICAL MAGAZINE NR. 27

ABOUT PROBLEM IX.33-RMM-24, SPRING EDITION 2020

By Marin Chirciu – Romania

1) Prove that in any acute-angled triangle the following inequality holds:

∑( )( )

.

/

Proposed by Marian Ursărescu – Romania

Solution

We prove the following lemma:

Lemma:

2) In the following inequality holds:

∑( )( )

( ) ( ) ( )

Proof.

Using the formula

, we obtain:

∑( )( )

.

/ .

/

∑( )( )

( ) ( ) ( )

, which follows from

∑( )( )

∑ ( )( )

and

∑ ( )( ) ( ) ( ) ( )

Let’s get to the main problem:

Using the Lemma it suffices to prove that:

( ) ( ) ( )

.

/

(1)

Using Gerretsen’s inequality and Euler’s inequality we have:

( ) ( ) ( )

, ( ) ( ) ( ) -

( ),( )( ) ( )

( ) -

Page 38: R. M. M. - 27...Tuzson, Florin Popovici, Viorel Gh. Vodă, Ovidiu Pop, E. Grosswald (USA), Leroy F. Meyers (USA), Francisco Bellot (Spain). Mihály encze contributed to ’’Gamma’’with

Romanian Mathematical Society-Mehedinți Branch 2020

37 ROMANIAN MATHEMATICAL MAGAZINE NR. 27

,( )( ) ( ) -

It follows that ∑( )( )

(2)

Using (2) in order to prove (1) it suffices to prove that:

, which follows from Mitrinovic’s inequality

.

It remains to prove that

, obviously from Euler’s inequality

. Equality holds if and only if the triangle is equilateral.

Remark: The inequality can be strengthened:

3) In the following relationship holds:

∑( )( )

.

/

Proposed by Marin Chirciu – Romania

Solution

Using (2) in order to prove the proposed inequality it suffices to prove that:

, which follows from Mitrinovic’s inequality

Equality holds if and only if the triangle is equilateral.

Remark.

Inequality 3) is stronger than inequality 1)

4) In the following inequality holds:

∑( )( )

.

/

.

/

Solution

See inequality 3) and .

/

.

/

, obviously from Euler’s inequality

. Equality holds if and only if the triangle is equilateral.

Remark.

If we replace with we propose:

5) In the following relationship holds:

Page 39: R. M. M. - 27...Tuzson, Florin Popovici, Viorel Gh. Vodă, Ovidiu Pop, E. Grosswald (USA), Leroy F. Meyers (USA), Francisco Bellot (Spain). Mihály encze contributed to ’’Gamma’’with

Romanian Mathematical Society-Mehedinți Branch 2020

38 ROMANIAN MATHEMATICAL MAGAZINE NR. 27

( )( )

4

5

Proposed by Marin Chirciu – Romania

Solution

We prove the following lemma:

Lemma:

6) In the following relationship holds:

∑( )( )

( ) ( )

Proof.

Using the formula

, we obtain:

∑( )( )

.

/ .

/

( )

( ) ( )

Let’s get back to the main problem.

The left hand inequality:

Using the Lemma it suffices to prove that: ( ) ( )

( ) ( ), which follows from Blundon-Gerretsen’s inequality

( )

( ). It remains to prove that:

( ) ( )

( )( )

( )( ) , obviously from Euler’s inequality .

Equality holds of and only if the triangle is equilateral.

The right hand inequality.

Using the Lemma it suffices to prove that: ( ) ( )

.

/

( ) ( ) , it follows from Gerretsen’s inequality

( )

. It follows to prove that:

( )

( ) ( ) (Euler’s inequality)

Equality holds if and only if the triangle is equilateral.

Page 40: R. M. M. - 27...Tuzson, Florin Popovici, Viorel Gh. Vodă, Ovidiu Pop, E. Grosswald (USA), Leroy F. Meyers (USA), Francisco Bellot (Spain). Mihály encze contributed to ’’Gamma’’with

Romanian Mathematical Society-Mehedinți Branch 2020

39 ROMANIAN MATHEMATICAL MAGAZINE NR. 27

PROPOSED PROBLEMS

5-CLASS-STANDARD

V.1. Find such that:

Proposed by Petre Stângescu-Romania

V.2. Find such that:

Proposed by Petre Stângescu-Romania

V.3. Let be . Find p- prime number such that ( ) ( )( )

Proposed by Petre Stângescu-Romania

V.4. Let be . If ( ) and find ( )( ) then find ( )( )

Proposed by Petre Stângescu-Romania

V.5. Find such that simultaneous are perfect square.

Proposed by Ștefan Marica-Romania

V.6. Find such that:

Proposed by Ștefan Marica-Romania

V.7. Let be a number formed from 2019 digits of . Prove that:

a) is not a perfect square. b) is not a perfect cube

c) is not a power having the oreder of a natural number bigger than .

Proposed by Ene Cristina, Nicu Elena – Romania

V.8. Prove that the number

Proposed by Iuliana Trașcă, Chiriță Aurel –Romania

V.9. Find the numbers having the form for which the number

is a perfect square. Proposed by Constantin Ionică – Romania

V.10. Find the natural nonzero numbers the divided to give quotient and the reminder

, and divided to give the quotient and the reminder .

Proposed by Constantin Ionică – Romania

Page 41: R. M. M. - 27...Tuzson, Florin Popovici, Viorel Gh. Vodă, Ovidiu Pop, E. Grosswald (USA), Leroy F. Meyers (USA), Francisco Bellot (Spain). Mihály encze contributed to ’’Gamma’’with

Romanian Mathematical Society-Mehedinți Branch 2020

40 ROMANIAN MATHEMATICAL MAGAZINE NR. 27

V.11. Dan has written on the board consecutive natural numbers and after that he deleted a

number. Find the deleted number, if the sum of the remained numbers is equal to .

Proposed by Constantin Ionică – Romania

V.12. Prove that is divisible with . Proposed by Daniel Stretcu – Romania

V.13. Solve in natural numbers the following equation: .

Proposed by Daniel Stretcu – Romania

V.14. Find the natural numbers having the following form , knowing that:

. Proposed by Daniel Stretcu – Romania

V.15. Prove that can be written as a sum of three perfect squares different and

nonzero, for any natural nonzero number. Proposed by Marin Chirciu – Romania

All solutions for proposed problems can be finded on the http//:www.ssmrmh.ro which is the adress of Romanian Mathematical

Magazine-Interactive Journal.

6-CLASS-STANDARD

VI.1. Solve for natural numbers: .

Proposed by Petre Stângescu-Romania

VI.2. If p-prime number then can not be a perfect cube.

Proposed by Petre Stângescu-Romania

VI.3. Find and -prime number such that: be a perfect square.

Proposed by Petre Stângescu-Romania

VI.4. Find - prime numbers; such that be a perfect square

. Proposed by Petre Stângescu-Romania

VI.5. Find such that: .

Proposed by Gheorghe Calafeteanu-Romania

Page 42: R. M. M. - 27...Tuzson, Florin Popovici, Viorel Gh. Vodă, Ovidiu Pop, E. Grosswald (USA), Leroy F. Meyers (USA), Francisco Bellot (Spain). Mihály encze contributed to ’’Gamma’’with

Romanian Mathematical Society-Mehedinți Branch 2020

41 ROMANIAN MATHEMATICAL MAGAZINE NR. 27

VI.6. Find such that

. Proposed by Gheorghe Calafeteanu-Romania

VI.7. Find a such that √ . Proposed by Ștefan Marica-Romania

VI.8. Find such that √ . Proposed by Ștefan Marica-Romania

VI.9. Find { }. Proposed by Ștefan Marica-Romania

VI.10. Let be the natural numbers, and ( ) ( )

. Prove that if

, then . Proposed by Constantin Ionică – Romania

VI.11. Find the pairs of nonzero natural numbers with for which

, - ( ) , where( ) is biggest common divisor and , - is the biggest

common divisor of the numbers and . Proposed by Constantin Ionică – Romania

VI.12. Find the numbers knowing that and are directly proportional with and ,

and are directly proportional with and , and .

Proposed by Constantin Ionică – Romania

VI.13. Prove that the number is divisible with , for any natural nonzero

number . Proposed by Daniel Stretcu – Romania

VI.14. Let be . Prove that . Proposed by Daniel Stretcu – Romania

VI.15. Prove that the number is divisible with 11.

Proposed by Daniel Stretcu – Romania

VI.16. Prove that it doesn’t exist the natural numbers such that

.

Proposed by Daniel Stretcu – Romania

VI.17. Find the natural numbers with three digits which divided to and give the

quota , respectively . Proposed by Daniel Stretcu – Romania

VI.18. Prove that: if and only if .

Proposed by Daniel Stretcu – Romania

VI.19. Find the quota of to the number .

Proposed by Daniel Stretcu – Romania

Page 43: R. M. M. - 27...Tuzson, Florin Popovici, Viorel Gh. Vodă, Ovidiu Pop, E. Grosswald (USA), Leroy F. Meyers (USA), Francisco Bellot (Spain). Mihály encze contributed to ’’Gamma’’with

Romanian Mathematical Society-Mehedinți Branch 2020

42 ROMANIAN MATHEMATICAL MAGAZINE NR. 27

VI.20. Prove that for any natural nonzero number , among the elements of the set

* + there is at least a power of .

Proposed by Daniel Stretcu – Romania

All solutions for proposed problems can be finded on the http//:www.ssmrmh.ro which is the adress of Romanian Mathematical

Magazine-Interactive Journal.

7-CLASS-STANDARD

VII.1. In obtuse ; ; . Find perimeter. Proposed by Petre Stângescu-Romania

VII.2. In ( )

. Find . Proposed by Petre Stângescu-Romania

VII.3. In ( ) . If ( √ )( )( ) then find ( ) ( ) . Proposed by Petre Stângescu-Romania VII.4 If in ( ) and ( ) then .

Proposed by Petre Stângescu-Romania

VII.5. In ( ) the following relationship holds:

(√ )

Proposed by Petre Stângescu-Romania

VII.6. In a bicentric issoscelles trapezian perimeter is . If is circumcentre

and ( ) find the sides of trapezian. Proposed by Gheorghe Calafeteanu-Romania

VII.7. In ( ) ( ) * + Prove that:

. Proposed by Gheorghe Calafeteanu-Romania

Page 44: R. M. M. - 27...Tuzson, Florin Popovici, Viorel Gh. Vodă, Ovidiu Pop, E. Grosswald (USA), Leroy F. Meyers (USA), Francisco Bellot (Spain). Mihály encze contributed to ’’Gamma’’with

Romanian Mathematical Society-Mehedinți Branch 2020

43 ROMANIAN MATHEMATICAL MAGAZINE NR. 27

VII.8. In . Find area and

semiperimeter. Proposed by Ștefan Marica-Romania VII.9. Solve for natural numbers:

{

√ √

√ √

√ √ √

Proposed by Ștefan Marica-Romania

VII.10. Solve for natural numbers: . Proposed by Ștefan Marica-Romania

VII.11 Find depending on (all natural numbers) such that:

( ) Proposed by Ștefan Marica-Romania

VII.12. Find the natural numbers with and prime numbers, knowing that:

√ √ √ Proposed by Constantin Ionică – Romania

VII.13. Find the natural nonzero numbers , knowing that:

and

.

Proposed by Constantin Ionică – Romania

VII.14. Find such that the equation:

has an unique solution. Proposed by Daniel Stretcu – Romania

VII.15. The real strictly positive numbers verify the relationship . Prove

that the number

is a perfect square natural number.

Proposed by Daniel Stretcu – Romania

VII.16. In a medical laboratory, a biologist finds that he has unitary cells of a

square, that are infected and then, after a time, the cells that have at least two neibours cells infected (having a common side) they also infect. Can the infection spread in all the square?

Proposed by Daniel Stretcu – Romania VII.17. A natural number formed from digits has the first digits . Prove that the

number cannot be a perfect square. Proposed by Daniel Stretcu – Romania

VII.18. Solve for in such that:

Proposed by Naren Bhandari-Nepal

Page 45: R. M. M. - 27...Tuzson, Florin Popovici, Viorel Gh. Vodă, Ovidiu Pop, E. Grosswald (USA), Leroy F. Meyers (USA), Francisco Bellot (Spain). Mihály encze contributed to ’’Gamma’’with

Romanian Mathematical Society-Mehedinți Branch 2020

44 ROMANIAN MATHEMATICAL MAGAZINE NR. 27

VII.19. Find the last digit of:

Proposed by Seyran Ibrahimov-Azerbaijan

All solutions for proposed problems can be finded on the http//:www.ssmrmh.ro which is the adress of Romanian Mathematical

Magazine-Interactive Journal.

8-CLASS-STANDARD

VIII.1. Solve for integers: ( )

Proposed by Petre Stângescu-Romania

VIII.2. Solve for integers:

Proposed by Petre Stângescu-Romania

VIII.3. Find: ⏟

( ). Proposed by Petre Stângescu-Romania

VIII.4. If then: . Find ⏟

( ).

Proposed by Petre Stângescu-Romania

VIII.5. Solve for natural numbers: ( ) ( ) .

Proposed by Ștefan Marica-Romania

VIII.6. Let be the set * +

a) Write the set in the form of an interval.

b) Find the natural number for which contains exactly integer numbers.

Proposed by Constantin Ionică – Romania

VIII.7. Let be the number , where .

a) Prove that can be written as a difference of two perfect squares.

b) If , prove that . Proposed by Constantin Ionică – Romania

Page 46: R. M. M. - 27...Tuzson, Florin Popovici, Viorel Gh. Vodă, Ovidiu Pop, E. Grosswald (USA), Leroy F. Meyers (USA), Francisco Bellot (Spain). Mihály encze contributed to ’’Gamma’’with

Romanian Mathematical Society-Mehedinți Branch 2020

45 ROMANIAN MATHEMATICAL MAGAZINE NR. 27

VIII.8. Let be positive real numbers such that * + . Prove that:

√ ( )

( )

( )

Proposed by Nguyen Viet Hung – Vietnam

VIII.9. Solve in real numbers the following equation:

√ √ √

Proposed by Nguyen Viet Hung – Vietnam

VIII.10. Let be non-negative real numbers such that . Prove that:

a) ( ) ( )

b) ( ) ( )

Proposed by Nguyen Viet Hung – Vietnam

VIII.11. Prove that if * + then:

( )

( )

( )

( )

Proposed by D.M. Bătinețu – Giurgiu,Alecu Orlando – Romania

VIII.12. Prove that if * + then:

( )

( )

( )

( )

Proposed by D.M. Bătinețu – Giurgiu,Cătălin Spiridon – Romania

VIII.13. Prove that if * + then:

( )

( )

( )

Proposed by D.M. Bătinețu – Giurgiu,Elena Alexie – Romania

VIII.14. If then:

Proposed by D.M. Bătinețu – Giurgiu,Gabriela Vasile – Romania

VIII.15. If then:

( )

( )( )

Proposed by D.M. Bătinețu – Giurgiu,Gheorghe Boroica – Romania

Page 47: R. M. M. - 27...Tuzson, Florin Popovici, Viorel Gh. Vodă, Ovidiu Pop, E. Grosswald (USA), Leroy F. Meyers (USA), Francisco Bellot (Spain). Mihály encze contributed to ’’Gamma’’with

Romanian Mathematical Society-Mehedinți Branch 2020

46 ROMANIAN MATHEMATICAL MAGAZINE NR. 27

VIII.16. If then:

( )( )( )( )

( ) ( ) √

Proposed by Daniel Sitaru – Romania

VIII.17. If are positive integers such that:

.

Find the unique solution for keeping as small as possible if it exists.

Proposed by Naren Bhandari-Nepal

VIII.18. If then:

( ) (

* (

* (

*

Proposed by Daniel Sitaru,Elena Iacob Meda – Romania

VIII.19. If then in the following relationship holds:

∑4

(

*

5

4 ( )

√ 5

Proposed by Daniel Sitaru,Alina Georgiana Ghiță – Romania VIII.20. If then:

4 ( )

5 ( )4

( )

5 ( )

Proposed by Daniel Sitaru – Romania VIII.21. Solve for natural numbers:

Proposed by Seyran Ibrahimov-Azerbaijan

VIII.22. Find such that:

Proposed by Seyran Ibrahimov-Azerbaijan

Page 48: R. M. M. - 27...Tuzson, Florin Popovici, Viorel Gh. Vodă, Ovidiu Pop, E. Grosswald (USA), Leroy F. Meyers (USA), Francisco Bellot (Spain). Mihály encze contributed to ’’Gamma’’with

Romanian Mathematical Society-Mehedinți Branch 2020

47 ROMANIAN MATHEMATICAL MAGAZINE NR. 27

VIII.23. If then:

Proposed by Seyran Ibrahimov-Azerbaijan

VIII.24. Solve:

4 , -

5

4 , -

5

Proposed by Seyran Ibrahimov-Azerbaijan

All solutions for proposed problems can be finded on the http//:www.ssmrmh.ro which is the adress of Romanian Mathematical

Magazine-Interactive Journal.

9-CLASS-STANDARD

IX.1. Let such that and . Prove that:

Proposed by Marin Chirciu – Romania

IX.2. Let be and such that . Prove that:

( )

( )

( )

( )

Proposed by Marin Chirciu – Romania

IX.3. Let , such that and . Prove that:

(

√ *

(

√ *

(

√ *

√( )

Proposed by Marin Chirciu – Romania

Page 49: R. M. M. - 27...Tuzson, Florin Popovici, Viorel Gh. Vodă, Ovidiu Pop, E. Grosswald (USA), Leroy F. Meyers (USA), Francisco Bellot (Spain). Mihály encze contributed to ’’Gamma’’with

Romanian Mathematical Society-Mehedinți Branch 2020

48 ROMANIAN MATHEMATICAL MAGAZINE NR. 27

IX.4. In the following relationship holds:

√ ( )

Proposed by Marin Chirciu – Romania

IX.5. In the following relationship holds:

, where

Proposed by Marin Chirciu – Romania

IX.6. Let be such that

, where . Prove:

.

/

.

/

.

/

, where .

Proposed by Marin Chirciu – Romania

IX.7. If then:

( )

Proposed by Marin Chirciu – Romania

IX.8. If then:

Proposed by Marin Chirciu – Romania

IX.9. If such that and , find the minimum of the expression:

Proposed by Marin Chirciu – Romania

IX.10. Let . Find the maximum of the expression:

√ , where is fixed.

Proposed by Marin Chirciu – Romania

IX.11. In the following relationship holds:

√ 4

5

Proposed by Marin Chirciu – Romania

IX.12. In the following relationship holds:

Page 50: R. M. M. - 27...Tuzson, Florin Popovici, Viorel Gh. Vodă, Ovidiu Pop, E. Grosswald (USA), Leroy F. Meyers (USA), Francisco Bellot (Spain). Mihály encze contributed to ’’Gamma’’with

Romanian Mathematical Society-Mehedinți Branch 2020

49 ROMANIAN MATHEMATICAL MAGAZINE NR. 27

4√

5

Proposed by Marin Chirciu – Romania

IX.13. Let be such that

. Prove that:

Proposed by Marin Chirciu – Romania

IX.14. Let such that and . Prove that:

√ √

Proposed by Marin Chirciu – Romania

IX.15. In the following relationship holds:

(

* (

* (

* ( )

Proposed by Marin Chirciu – Romania

IX.16. Let such that and

. Prove that:

Proposed by Marin Chirciu – Romania

IX.17. Let be such that and . Prove that:

Proposed by Marin Chirciu – Romania

IX.18. Let . Prove that:

∑4

5

( )

Proposed by Marin Chirciu – Romania

IX.19. Prove that in any triangle with the standard notations:

( ) ( )

Proposed by Nguyen Viet Hung – Vietnam

IX.20. Prove that in any triangle ,

Page 51: R. M. M. - 27...Tuzson, Florin Popovici, Viorel Gh. Vodă, Ovidiu Pop, E. Grosswald (USA), Leroy F. Meyers (USA), Francisco Bellot (Spain). Mihály encze contributed to ’’Gamma’’with

Romanian Mathematical Society-Mehedinți Branch 2020

50 ROMANIAN MATHEMATICAL MAGAZINE NR. 27

( )

Proposed by Nguyen Viet Hung – Vietnam

IX.21. In – orthocenter the following relationship holds:

( )

Proposed by Marian Ursărescu – Romania

IX.22. If in ( ) ( ) ( ) * + then:

Proposed by Marian Ursărescu – Romania

IX.23. Let be the circumcevian triangle of Nagel’s point of . Prove that:

, -

, - (

*

Proposed by Marian Ursărescu – Romania

IX.24. In the following relationship holds:

(

* (

* (

* √ .

/

Proposed by Marian Ursărescu – Romania

IX.25. In the following relationship holds:

( )

(

*

Proposed by Marian Ursărescu – Romania

IX.26. In the following relationship holds:

( √ ) ( √ )

( √ )

Proposed by D.M. Bătinețu – Giurgiu,Dan Grigorie – Romania

IX.27. If then:

√( )( )

√( )( )

√( )( )

Proposed by D.M. Bătinețu – Giurgiu,Gheorghe Boroica – Romania

IX.28. If then:

Page 52: R. M. M. - 27...Tuzson, Florin Popovici, Viorel Gh. Vodă, Ovidiu Pop, E. Grosswald (USA), Leroy F. Meyers (USA), Francisco Bellot (Spain). Mihály encze contributed to ’’Gamma’’with

Romanian Mathematical Society-Mehedinți Branch 2020

51 ROMANIAN MATHEMATICAL MAGAZINE NR. 27

∑( )

Proposed by D.M. Bătinețu – Giurgiu,Nicolae Radu – Romania

IX.29. In the following relationship holds:

Proposed by D.M. Bătinețu – Giurgiu,Mihaela Dăianu – Romania

IX.30. In the following relationship holds:

Proposed by D.M. Bătinețu – Giurgiu,Luiza Cremeneanu – Romania

IX.31. If then:

∑.

/

∑.

/

Proposed by D.M. Bătinețu – Giurgiu,Neculai Stanciu – Romania

IX.32. If then:

√ ( √ √ )

Proposed by D.M. Bătinețu – Giurgiu,Neculai Stanciu – Romania

IX.33. If then:

4

5 4

54

5 ∏( )

Proposed by D.M. Bătinețu – Giurgiu,Alina Tigae – Romania

IX.34. If √ √ √ then:

(

√ *

(

√ *

(

√ *

Proposed by D.M. Bătinețu – Giurgiu,Amelia Curcă Năstăselu– Romania

IX.35. If then:

Proposed by D.M. Bătinețu – Giurgiu,Camelia Dană – Romania

IX.36. If * + then:

Page 53: R. M. M. - 27...Tuzson, Florin Popovici, Viorel Gh. Vodă, Ovidiu Pop, E. Grosswald (USA), Leroy F. Meyers (USA), Francisco Bellot (Spain). Mihály encze contributed to ’’Gamma’’with

Romanian Mathematical Society-Mehedinți Branch 2020

52 ROMANIAN MATHEMATICAL MAGAZINE NR. 27

∑.

/

∑(

*

Proposed by D.M. Bătinețu – Giurgiu,Patricia Anicuța Bețiu – Romania

IX.37. If then in the following relationship holds:

:∑

;:∑

( )

;

( )

Proposed by D.M. Bătinețu – Giurgiu,Gheorghe Stoica – Romania

IX.38. If then in the following relationship holds:

∑( )

Proposed by D.M. Bătinețu – Giurgiu,Nicolae Mușuroia – Romania

IX.39. If then in the following relationship holds:

( )

( )

( )

Proposed by D.M. Bătinețu – Giurgiu,Cătălin Pană – Romania

IX.40. If then in the following relationship holds:

∑ ( ) ( )

Proposed by D.M. Bătinețu – Giurgiu,Gheorghe Boroica – Romania

IX.41. If then in the following relationship holds: ( )

( )

( )

Proposed by D.M. Bătinețu – Giurgiu,Claudiu Ciulcu – Romania

IX.42. If then in the following relationship holds: ( )

( )

( )

Proposed by D.M. Bătinețu – Giurgiu,Roxana Vasile – Romania

IX.43. In Nagel’s cevian, the following relationship holds:

:√

√ ;∑√

Proposed by Bogdan Fuștei – Romania

Page 54: R. M. M. - 27...Tuzson, Florin Popovici, Viorel Gh. Vodă, Ovidiu Pop, E. Grosswald (USA), Leroy F. Meyers (USA), Francisco Bellot (Spain). Mihály encze contributed to ’’Gamma’’with

Romanian Mathematical Society-Mehedinți Branch 2020

53 ROMANIAN MATHEMATICAL MAGAZINE NR. 27

IX.44. In – Nagel’s cevian the following relationship holds:

√ ∑

∑√

Proposed by Bogdan Fuștei – Romania

IX.45. In – Nagel’s cevian, – Gergonne’s cevian the following relationship holds:

∑ ( √ )

– area , -

Proposed by Bogdan Fuștei – Romania

IX.46. In – incenter, the following relationship holds:

:∑( )

;:∑( )

; √ :∑( )

;

Proposed by Bogdan Fuștei – Romania

IX.47. In – Nagel’s cevian, – Gergonne’s cevian the following relationship holds:

(

* ∑4

5

Proposed by Bogdan Fuștei – Romania

IX.48. In – Gergonne’s cevian the following relationship holds:

∑( )

Proposed by Bogdan Fuștei – Romania IX.49. In – Nagel’s cevian the following relationship holds:

∑ √

√ (

*

Proposed by Bogdan Fuștei – Romania IX.50. In the following relationship holds:

Proposed by Bogdan Fuștei – Romania IX.51. In – Nagel’s cevian, – Gergonne’s cevian the following relationship holds:

(

* ∑

Proposed by Bogdan Fuștei – Romania

IX.52. In the following relationship holds:

Page 55: R. M. M. - 27...Tuzson, Florin Popovici, Viorel Gh. Vodă, Ovidiu Pop, E. Grosswald (USA), Leroy F. Meyers (USA), Francisco Bellot (Spain). Mihály encze contributed to ’’Gamma’’with

Romanian Mathematical Society-Mehedinți Branch 2020

54 ROMANIAN MATHEMATICAL MAGAZINE NR. 27

( √ )

Proposed by Bogdan Fuștei – Romania

IX.53. In the following relationship holds:

:∑

;:∑

;

Proposed by Bogdan Fuștei – Romania

IX.54. In – Nagel’s cevian, the following relationship holds:

(

*

Proposed by Bogdan Fuștei – Romania

IX.55. In the following relationship holds:

∏( )

Proposed by Bogdan Fuștei – Romania

IX.56. If then:

( )

( )

Proposed by Daniel Sitaru,Delia Popescu – Romania

IX.57.

( )

( )

( )

( )

( )

( )

( )

( )

( ) ( )

Prove that:

Proposed by Daniel Sitaru,Delia Schneider – Romania

IX.58. Let be a triangle and the lengths of the medians. Then, the following

inequality holds:

, where is the semiperimeter of . When do we have

equality?

Proposed by Gheorghe Alexe and George – Florin Șerban– Romania

Page 56: R. M. M. - 27...Tuzson, Florin Popovici, Viorel Gh. Vodă, Ovidiu Pop, E. Grosswald (USA), Leroy F. Meyers (USA), Francisco Bellot (Spain). Mihály encze contributed to ’’Gamma’’with

Romanian Mathematical Society-Mehedinți Branch 2020

55 ROMANIAN MATHEMATICAL MAGAZINE NR. 27

IX.59. Find the positive numbers and which satisfy the relationship:

( )( )( )

Proposed by Gheorghe Alexe and George – Florin Șerban– Romania

IX.60. If , with . Prove the following inequality:

Proposed by Gheorghe Alexe and George – Florin Șerban– Romania

IX.61. Prove that in any triangle with the sides with the following

inequality holds: ( )

( )

( )

( )

Proposed by Gheorghe Alexe and George – Florin Șerban– Romania

IX.62. Prove that in any triangle with the sides the following inequality holds: ( )

( )

( )

( ) ( )

( )

Proposed by Gheorghe Alexe and George – Florin Șerban– Romania

IX.64. If in then:

Proposed by Daniel Sitaru,Dorina Goiceanu – Romania

IX.65. Applications of Tarek’s Lemmas. In – Nagel’s cevians. Prove that:

Proposed by Daniel Sitaru,Constantina Prunaru – Romania

IX.66. In – Gergonne’s cevians, – Nagel’s cevians. Prove that:

Proposed by Daniel Sitaru – Romania

IX.67. In the following relationship holds: √ √ √ √

Proposed by Daniel Sitaru – Romania

IX.68. If in √ then: √

Proposed by Daniel Sitaru – Romania

IX.69. If then:

Page 57: R. M. M. - 27...Tuzson, Florin Popovici, Viorel Gh. Vodă, Ovidiu Pop, E. Grosswald (USA), Leroy F. Meyers (USA), Francisco Bellot (Spain). Mihály encze contributed to ’’Gamma’’with

Romanian Mathematical Society-Mehedinți Branch 2020

56 ROMANIAN MATHEMATICAL MAGAZINE NR. 27

( )

Proposed by Daniel Sitaru – Romania

IX.70. Prove that in any the following inequality holds:

( )

( )

( )

√ ( )√

Proposed by Marin Chirciu – Romania

IX.71. If with and , prove that:

Proposed by Marin Chirciu – Romania

IX.72. If and prove that:

Proposed by Marin Chirciu – Romania

IX.73. Solve in the following equation:

( )√ √ ( ) √ , where

Proposed by Marin Chirciu – Romania

IX.74. Let be , such that and . Prove that:

Proposed by Marin Chirciu – Romania

IX.75. Let be , such that and . Prove that:

( )

( )

( )

Proposed by Marin Chirciu – Romania

IX.76. Let be , such that and

. Prove that:

Proposed by Marin Chirciu – Romania

IX.77. Let be , such that and . Prove that:

( )

( )

( ) (

*

Proposed by Marin Chirciu – Romania

Page 58: R. M. M. - 27...Tuzson, Florin Popovici, Viorel Gh. Vodă, Ovidiu Pop, E. Grosswald (USA), Leroy F. Meyers (USA), Francisco Bellot (Spain). Mihály encze contributed to ’’Gamma’’with

Romanian Mathematical Society-Mehedinți Branch 2020

57 ROMANIAN MATHEMATICAL MAGAZINE NR. 27

IX.78. In the following relationship hold: √∑ ∑ ∑

Proposed by Marin Chirciu – Romania

IX.79. Let be and

. Prove that:

Proposed by Marin Chirciu – Romania

IX.80. Let be . Solve in real numbers the system of equations: {

( )

( )

( )

Proposed by Marin Chirciu – Romania

IX.81. In the following relationship holds:

∑(

*

∑(

*

Proposed by Daniel Sitaru – Romania

IX.82. Find such that:

Proposed by Seyran Ibrahimov-Azerbaijan

IX.83. If then:

Proposed by Seyran Ibrahimov-Azerbaijan

All solutions for proposed problems can be finded on the http//:www.ssmrmh.ro which is the adress of Romanian Mathematical

Magazine-Interactive Journal.

10-CLASS-STANDARD

Page 59: R. M. M. - 27...Tuzson, Florin Popovici, Viorel Gh. Vodă, Ovidiu Pop, E. Grosswald (USA), Leroy F. Meyers (USA), Francisco Bellot (Spain). Mihály encze contributed to ’’Gamma’’with

Romanian Mathematical Society-Mehedinți Branch 2020

58 ROMANIAN MATHEMATICAL MAGAZINE NR. 27

X.1. Solve in the following equations:

a) √

b) √

c) ( ) √

.

Proposed by Lucian Tuțescu, Mirea Mihaela Mioara – Romania

X.2. Let be the sequence of real numbers ( ) having the property that

( ) ( ) . Prove that .

Proposed by Cristina Ene, Elena Nicu– Romania

X.3. Let be . Solve the following equation:

( )

Proposed by Marin Chirciu – Romania

X.4. In the following relationship holds:

( ) , where

Proposed by Marin Chirciu – Romania

X.5. Let be . Prove that:

Proposed by Marin Chirciu – Romania

X.6. In the following relationship holds:

( )

Proposed by Marin Chirciu – Romania

X.7. Let be positive real numbers such that and . Prove that:

Proposed by Marin Chirciu – Romania

X.8. In the following relationship holds:

∑ ∑

, where .

Proposed by Marin Chirciu – Romania

X.9. In the following relationship holds:

Page 60: R. M. M. - 27...Tuzson, Florin Popovici, Viorel Gh. Vodă, Ovidiu Pop, E. Grosswald (USA), Leroy F. Meyers (USA), Francisco Bellot (Spain). Mihály encze contributed to ’’Gamma’’with

Romanian Mathematical Society-Mehedinți Branch 2020

59 ROMANIAN MATHEMATICAL MAGAZINE NR. 27

Proposed by Marin Chirciu – Romania

X.10. In the following relationship holds:

, where

Proposed by Marin Chirciu – Romania

X.11. Let such that . Prove that:

( )

( )

( )

Proposed by Marin Chirciu – Romania

X.12. In the following relationship holds:

( )

.

/ ∑

( )( )

, where

Proposed by Marin Chirciu – Romania

X.13 Let be the inradii of circumcevian triangle of incenter in acute . Prove that:

Proposed by Marian Ursărescu – Romania

X.14. In the following relationship holds:

√ 4 (

*

5

Proposed by Marian Ursărescu – Romania

X.15. Let be the excentral triangle of . Prove that:

Proposed by Marian Ursărescu – Romania

X.16. – centroid, – Lemoine’s point. Prove that:

, -

, - .

/

Proposed by Marian Ursărescu – Romania

Page 61: R. M. M. - 27...Tuzson, Florin Popovici, Viorel Gh. Vodă, Ovidiu Pop, E. Grosswald (USA), Leroy F. Meyers (USA), Francisco Bellot (Spain). Mihály encze contributed to ’’Gamma’’with

Romanian Mathematical Society-Mehedinți Branch 2020

60 ROMANIAN MATHEMATICAL MAGAZINE NR. 27

X.17. In – Nagel’s point ( ) ( )

( ). Prove that:

Proposed by Marian Ursărescu – Romania

X.18. Let be the circumcevian triangle of Lemonine’s point of .

Prove that:

(

*

, -

, - (

*

Proposed by Marian Ursărescu – Romania

X.19. In – orthocenter the following relationship holds:

( )

Proposed by Marian Ursărescu – Romania

X.20. In the following relationship holds:

Proposed by D.M. Bătinețu – Giurgiu,Gheorge Stoica – Romania

X.21. If then ( ) : √ √

Proposed by D.M. Bătinețu – Giurgiu,Gheorghe Stoica – Romania

X.22. If then:

Proposed by D.M. Bătinețu – Giurgiu,Virginia Grigorescu – Romania

X.23. In the following relationship holds:

( )

( )

( )

Proposed by D.M. Bătinețu – Giurgiu,Neculai Stanciu – Romania

X.24. In the following relationship holds:

Proposed by D.M. Bătinețu – Giurgiu,Dan Nănuți – Romania

X.25. If then in the following relationship holds:

( )

( )

( )

( ) ( )

Proposed by D.M. Bătinețu – Giurgiu,Claudia Nănuți – Romania

Page 62: R. M. M. - 27...Tuzson, Florin Popovici, Viorel Gh. Vodă, Ovidiu Pop, E. Grosswald (USA), Leroy F. Meyers (USA), Francisco Bellot (Spain). Mihály encze contributed to ’’Gamma’’with

Romanian Mathematical Society-Mehedinți Branch 2020

61 ROMANIAN MATHEMATICAL MAGAZINE NR. 27

X.26. If then in the following relationship holds:

4

5

4

5

4

5

.

/

Proposed by D.M. Bătinețu – Giurgiu,Ștefan Marica – Romania

X.27. If then in the following relationship holds:

( )

( )( )

Proposed by D.M. Bătinețu – Giurgiu,Cristina Spiridon – Romania

X.28. In – centroid;

the following relationship

holds:

Proposed by D.M. Bătinețu – Giurgiu – Romania

X.29. If then in the following relationship holds:

Proposed by D.M. Bătinețu – Giurgiu,Lavinia Trincu– Romania

X.30. If then in the following relationship holds:

( )

Proposed by D.M. Bătinețu – Giurgiu,Mihaela Nascu – Romania

X.31. If then in the following relationship holds:

∑( ) ( ) ( )

Proposed by D.M. Bătinețu – Giurgiu,Mihaela Stăncele – Romania

X.32. If then in the following relationship holds:

Proposed by D.M. Bătinețu – Giurgiu,Nicolae Mușuroia – Romania

X.33. If denote √

;

then the following

relationship holds:

Proposed by D.M. Bătinețu – Giurgiu,Ileana Duma – Romania

Page 63: R. M. M. - 27...Tuzson, Florin Popovici, Viorel Gh. Vodă, Ovidiu Pop, E. Grosswald (USA), Leroy F. Meyers (USA), Francisco Bellot (Spain). Mihály encze contributed to ’’Gamma’’with

Romanian Mathematical Society-Mehedinți Branch 2020

62 ROMANIAN MATHEMATICAL MAGAZINE NR. 27

X.34. If then the following relationship holds in :

√ ( )

Proposed by D.M. Bătinețu – Giurgiu,Ileana Stanciu – Romania

X.35. If then in the following relationship holds:

Proposed by D.M. Bătinețu – Giurgiu,Nicolae Mușuroia – Romania

X.36. If then in the following relationship holds:

( ) √

Proposed by D.M. Bătinețu – Giurgiu,Daniela Beldea – Romania

X.37. If , different in pairs, ,

( ) ( ) ( ) ∑

Prove that: .

Proposed by Marian Ursărescu – Romania

X.38 If –internal bisectors then:

Proposed by Marian Ursărescu – Romania

X.39 In acute – orthocenter, – altitudes. Prove that:

( ) ( )

Proposed by Marian Ursărescu – Romania

X.40. Prove that it doesn’t exists ( ) such that: ( )

Proposed by Marian Ursărescu – Romania

X.41. In Brocard angle, the following relationship holds:

( ) ( ) ( )

Proposed by Marian Ursărescu – Romania

X.42. , are different in pairs,

Page 64: R. M. M. - 27...Tuzson, Florin Popovici, Viorel Gh. Vodă, Ovidiu Pop, E. Grosswald (USA), Leroy F. Meyers (USA), Francisco Bellot (Spain). Mihály encze contributed to ’’Gamma’’with

Romanian Mathematical Society-Mehedinți Branch 2020

63 ROMANIAN MATHEMATICAL MAGAZINE NR. 27

( ) ( ) ( ) ∑

Prove that: .

Proposed by Marian Ursărescu – Romania

X.43. In acute the following relatioinship holds:

( ) ( )

Proposed by Marian Ursărescu – Romania

X.44. In acute the following relationship holds:

4 (

*

5

Proposed by Marian Ursărescu – Romania

X.45. Let be the median triangle of . If ( ) ( ) ( ) collinears,

– incenter. Prove that:

( )

( )

( )

Proposed by Marian Ursărescu – Romania

X.46. In – internal bisectors, ( ) ( ) ( ).

Prove that: , - .

/ , - , -

Proposed by Marian Ursărescu – Romania

X.47. Let be the inradii of circumcevian triangle of incenter in acute . Prove that:

( )

Proposed by Marian Ursărescu – Romania

X.48. In the following relationship holds:

( )

Proposed by Marian Ursărescu – Romania

X.49. In – Brocard’s angle, the following relationship holds:

√ (

√ (

*+

Proposed by Bogdan Fuștei – Romania

Page 65: R. M. M. - 27...Tuzson, Florin Popovici, Viorel Gh. Vodă, Ovidiu Pop, E. Grosswald (USA), Leroy F. Meyers (USA), Francisco Bellot (Spain). Mihály encze contributed to ’’Gamma’’with

Romanian Mathematical Society-Mehedinți Branch 2020

64 ROMANIAN MATHEMATICAL MAGAZINE NR. 27

X.50. In – incenter, – Nagel’s point, the following relationship holds:

(

*∑

Proposed by Bogdan Fuștei – Romania

X.51. In – Nagel’s cevian, – Gergonne’s cevian, the following relationship holds:

√ √

Proposed by Bogdan Fuștei – Romania

X.52. In any triangle we have:

√ ∑ √

Proposed by Bogdan Fuștei – Romania

X.53. In – incenter, – Nagel’s point, – Nagel’s cevian the following

relationship holds:

Proposed by Bogdan Fuștei – Romania

X.54. If in – nine – point center then: √ √ √ √

Proposed by Daniel Sitaru,Nineta Oprescu – Romania

X.55. If then:

( ) ( ) ( )

Proposed by Daniel Sitaru,Claudia Nănuți – Romania

X.56. If

then:

Proposed by Daniel Sitaru,Claudia Nănuți – Romania

X.57. If then:

Page 66: R. M. M. - 27...Tuzson, Florin Popovici, Viorel Gh. Vodă, Ovidiu Pop, E. Grosswald (USA), Leroy F. Meyers (USA), Francisco Bellot (Spain). Mihály encze contributed to ’’Gamma’’with

Romanian Mathematical Society-Mehedinți Branch 2020

65 ROMANIAN MATHEMATICAL MAGAZINE NR. 27

∑(

.

/ ∑.

/

,

Proposed by Daniel Sitaru,Claudia Nănuți – Romania

X.58. If then:

Proposed by Daniel Sitaru – Romania

X.59. If then:

4( )√ .

/√

5

:√( ) √.

/

;

Proposed by Daniel Sitaru – Romania

X.60. If then:

( √ ) √

( √ ) √

( √ ) √

Proposed by Daniel Sitaru,Mădălina Giurgescu – Romania

X.61. Solve for real numbers: √

√ √

Proposed by Daniel Sitaru,Mădălina Giurgescu– Romania

X.62. If then: ( √ √ ) .

√ /

When does equality holds?

Proposed by Daniel Sitaru,Laura Zaharia – Romania

X.63. If then:

√ √

√ √

√ √

Proposed by Daniel Sitaru – Romania

X.64. If sides in a cyclic quadrilateral with – area, – semiperimeter then:

.

/(

*(

* (

* (√ )

Proposed by Daniel Sitaru – Romania

Page 67: R. M. M. - 27...Tuzson, Florin Popovici, Viorel Gh. Vodă, Ovidiu Pop, E. Grosswald (USA), Leroy F. Meyers (USA), Francisco Bellot (Spain). Mihály encze contributed to ’’Gamma’’with

Romanian Mathematical Society-Mehedinți Branch 2020

66 ROMANIAN MATHEMATICAL MAGAZINE NR. 27

X.65. If in ( ) then: ( )( )( )

Proposed by Daniel Sitaru – Romania

X.66. , - – pyramid, ( ) ( ) ( )

( ) ( ) ( ) – radii of circumsphere, – radii of insphere. Prove that:

∑(

*

Proposed by Daniel Sitaru – Romania

X.67.

Prove that:

Proposed by Daniel Sitaru,Tatiana Cristea – Romania

X.68. Prove that: .

/ is divisible by * +

Proposed by Prem Kumar-India

X.69. In – Bevan’s point, – excenters, ( )

( ) ( ) the following relationship holds:

( )

Proposed by Mehmet Sahin-Turkey

X.70. In – Bevan’s point, – excenters, ( ),

( ) ( ) the following relationship holds:

(, - , -)

Proposed by Mehmet Sahin-Turkey

X.71. In – Bevan’s point, – excenters, ( ),

( ) ( ) – area, the following relationship holds:

(

*

( )

Proposed by Mehmet Sahin-Turkey

Page 68: R. M. M. - 27...Tuzson, Florin Popovici, Viorel Gh. Vodă, Ovidiu Pop, E. Grosswald (USA), Leroy F. Meyers (USA), Francisco Bellot (Spain). Mihály encze contributed to ’’Gamma’’with

Romanian Mathematical Society-Mehedinți Branch 2020

67 ROMANIAN MATHEMATICAL MAGAZINE NR. 27

X.72. In – excenters the following relationship holds:

(

*

(

*

(

*

( )

Proposed by Mehmet Sahin-Turkey

X.73. In – Bevan’s point, – excenters, ( )

( ) ( ) – area, the following relationship holds:

( )

( )

( )

( )

Proposed by Mehmet Sahin-Turkey

X.74. Find that in any triangle the following inequality holds: ∑

When does the equality holds?

Proposed by Gheorghe Alexe and George – Florin Șerban-Romania

X.75. Prove that in any triangle the following inequality holds:

Proposed by Gheorghe Alexe and George – Florin Șerban-Romania

X.76. * + different in pairs ( ) ( ) ( ). If

(

)

then .

Proposed by Marian Ursărescu – Romania

X.77. Let be the intouch triangle of . Let be the orthic triangle of

. Prove that: [ ]

, -

Proposed by Marian Ursărescu – Romania

X.78. In acute denote – radii of circle tangent simultaneous to incircle and to sides

. Define analogous . Prove that:

Proposed by Marian Ursărescu – Romania

X.79. * +, different in pairs,

( ) ( ) ( ). Prove that:

Page 69: R. M. M. - 27...Tuzson, Florin Popovici, Viorel Gh. Vodă, Ovidiu Pop, E. Grosswald (USA), Leroy F. Meyers (USA), Francisco Bellot (Spain). Mihály encze contributed to ’’Gamma’’with

Romanian Mathematical Society-Mehedinți Branch 2020

68 ROMANIAN MATHEMATICAL MAGAZINE NR. 27

∑ ( )

( )

( ) ( ) ( )

Proposed by Marian Ursărescu – Romania

X.80. In acute – inradii of orthic triangle. Prove that:

Proposed by Marian Ursărescu – Romania

X.81. In acute – Lemoine’s point the following relationship holds:

( ) (

*

Proposed by Marian Ursărescu – Romania

X.82. In acute the following relationship holds:

Proposed by Marian Ursărescu – Romania

X.83. If , different in pairs, ,

( ) ( ) ( ) ∑

. Prove that: .

Proposed by Marian Ursărescu – Romania

X.84. In – Brocard’s point the following relationship holds:

( ) ( ) ( )

Proposed by Marian Ursărescu – Romania

X.85. In acute – orthocenter, – altitudes. Prove that:

( ) ( ) ( )

Proposed by Marian Ursărescu – Romania

X.86. * + different in pairs ( ) ( ) ( )

( )

( )

( )

. Prove that: .

Proposed by Marian Ursărescu – Romania

X.87. In acute – Lemoine’s point, – bisectors of

( ) ( ) ( ). Prove that: , -

, - .

/

Proposed by Marian Ursărescu – Romania

X.88. * +, different in pairs,

Page 70: R. M. M. - 27...Tuzson, Florin Popovici, Viorel Gh. Vodă, Ovidiu Pop, E. Grosswald (USA), Leroy F. Meyers (USA), Francisco Bellot (Spain). Mihály encze contributed to ’’Gamma’’with

Romanian Mathematical Society-Mehedinți Branch 2020

69 ROMANIAN MATHEMATICAL MAGAZINE NR. 27

. Find:

Proposed by Marian Ursărescu – Romania

X.89. , different in pairs, ; ( ) ( ) ( )

Proposed by Marian Ursărescu – Romania

X.90. In – internal bisectors, ( ) ( ) ( ).

Prove that: , - .

/ , - , -

Proposed by Marian Ursărescu – Romania

X.91. In acute the following relationship holds:

∑ (

*

Proposed by Marian Ursărescu – Romania

X.92. In – centroid, – symedians in

( ) ( ) ( ). Prove that: , -

, - .

/

Proposed by Marian Ursărescu – Romania

X.93. * +, different in pairs, ( ) ( ) ( )

( ) √ ( ) ( ) ( )

Proposed by Marian Ursărescu – Romania

X.94. , different in pairs, ( ) ( ) ( )

∑ ( )

Proposed by Marian Ursărescu – Romania

X.95. If in – Nagel’s cevians then:

Proposed by Daniel Sitaru – Romania

X.96. If in acute – incentre, – centroid, – orthocenter, – Nagel’s point then:

Proposed by Daniel Sitaru,Iulia Sanda – Romania

Page 71: R. M. M. - 27...Tuzson, Florin Popovici, Viorel Gh. Vodă, Ovidiu Pop, E. Grosswald (USA), Leroy F. Meyers (USA), Francisco Bellot (Spain). Mihály encze contributed to ’’Gamma’’with

Romanian Mathematical Society-Mehedinți Branch 2020

70 ROMANIAN MATHEMATICAL MAGAZINE NR. 27

X.97. In – Nagel’s cevians, – Gergonne’s cevians. Find .

Proposed by Daniel Sitaru – Romania

X.98. If then: ( ) ( ) ( ) ( )

Proposed by Daniel Sitaru – Romania

X.99. Prove that in any triangle the following inequality holds:

(

*

Proposed by Marin Chirciu – Romania

X.100. Prove that in any triangle the following relationship holds:

Proposed by Marin Chirciu – Romania

X.101. Let be , such that . Prove that:

, where is a natural number.

Proposed by Marin Chirciu – Romania

X.102. Let be , such that . Prove that:

, where is a natural number. Proposed by Marin Chirciu – Romania

X.103. In the following relationship holds:

.

/

.

/

.

/

Proposed by Marin Chirciu – Romania

All solutions for proposed problems can be finded on the http//:www.ssmrmh.ro which is the adress of Romanian Mathematical

Magazine-Interactive Journal.

11-CLASS-STANDARD

Page 72: R. M. M. - 27...Tuzson, Florin Popovici, Viorel Gh. Vodă, Ovidiu Pop, E. Grosswald (USA), Leroy F. Meyers (USA), Francisco Bellot (Spain). Mihály encze contributed to ’’Gamma’’with

Romanian Mathematical Society-Mehedinți Branch 2020

71 ROMANIAN MATHEMATICAL MAGAZINE NR. 27

XI.1. ( ) ( ( ) ) . Find:

( ) (( ) )

Proposed by Marian Ursărescu – Romania

XI.2. Find all functions such that:

( ) ( ) ( ) ( )

Proposed by Marian Ursărescu – Romania

XI.3. ( ) . Find: ( )

Proposed by Marian Ursărescu – Romania

XI.4 , - ( ) ( ) – continuous. Prove that:

, - ( ) ( ) √ ( )

Proposed by Marian Ursărescu – Romania

XI.5. Find ( ) such that: (

+

Proposed by Marian Ursărescu – Romania

XI.6. ( ) (

+. Find: ( ) ( )

Proposed by Marian Ursărescu – Romania

XI.7. If , then:

∑ ( ) ( )

( ) ( ) ( )

Proposed by Florică Anastase – Romania

XI.8. If , then:

Proposed by Florică Anastase – Romania

XI.9. If , then: ( ) .

/ ( )

Proposed by Florică Anastase – Romania

XI.10. If ∑

(

) , find:

.

/

Proposed by Florică Anastase – Romania

Page 73: R. M. M. - 27...Tuzson, Florin Popovici, Viorel Gh. Vodă, Ovidiu Pop, E. Grosswald (USA), Leroy F. Meyers (USA), Francisco Bellot (Spain). Mihály encze contributed to ’’Gamma’’with

Romanian Mathematical Society-Mehedinți Branch 2020

72 ROMANIAN MATHEMATICAL MAGAZINE NR. 27

XI.11. If .

/, then:

. ( )

/

. ( )

/

( ( )) ( ( ))

Proposed by Florică Anastase – Romania XI.12. If , then:

∏4

5

(∏

+

Proposed by Florică Anastase – Romania XI.13. If ( ) or ( ) , then:

√:∏

; :∑

;

(

√(∑

) (∑ ( ) )

)

Proposed by Florică Anastase – Romania XI.14. If ( - then prove:

∑( )( )

( )( ( )( ))

Proposed by Florică Anastase – Romania

XI.15. If ( ) ( ) ( ) are sequences of real numbers such that:

∑.

/

∑.

/ (

*

(

( )

( )

( )*

Find: ( )

Proposed by Florică Anastase – Romania

XI.16. If ( ) then:

∑ 4( )( )

√ 5

√∏ ( ) ∑ ( )

Proposed by Florică Anastase – Romania

XI.17. If ,

( )

:

;

then: ( ) ( ) ( ) √

Proposed by Daniel Sitaru – Romania

XI.18. If then:

Page 74: R. M. M. - 27...Tuzson, Florin Popovici, Viorel Gh. Vodă, Ovidiu Pop, E. Grosswald (USA), Leroy F. Meyers (USA), Francisco Bellot (Spain). Mihály encze contributed to ’’Gamma’’with

Romanian Mathematical Society-Mehedinți Branch 2020

73 ROMANIAN MATHEMATICAL MAGAZINE NR. 27

|

| |

| ( )

|

|

Proposed by Daniel Sitaru,Doina Cristina Călina – Romania

XI.19. If .

/ then:

( ) ( ) ( ) √

Proposed by Daniel Sitaru – Romania

XI.20. If

then: .

( )

/ .

( )

/ .

( )

/ .

( )

/

Proposed by Daniel Sitaru – Romania

XI.21 If then: .

/ .

/ .

/

Proposed by Daniel Sitaru,Corina Ionescu – Romania

XI.22. Find all continuous function satisfying: ( ) ( ( )) and

( ) for all real and real .

Proposed by Prem Kumar-India

XI.23. Prove that:

Proposed by Prem Kumar-India

XI.24. Let ( ) ( )

* +. Prove that if is prime then:

.

/ ( ) for

Proposed by Prem Kumar-India

XI.25. Find:

(

+

Proposed by Prem Kumar-India

Page 75: R. M. M. - 27...Tuzson, Florin Popovici, Viorel Gh. Vodă, Ovidiu Pop, E. Grosswald (USA), Leroy F. Meyers (USA), Francisco Bellot (Spain). Mihály encze contributed to ’’Gamma’’with

Romanian Mathematical Society-Mehedinți Branch 2020

74 ROMANIAN MATHEMATICAL MAGAZINE NR. 27

XI.26. Consider the following function defined for all real numbers . ( )

How many integers are there in the range of ?

Proposed by Prem Kumar-India

XI.27. Prove that .

/ is odd iff is a non-negative integral power of .

Proposed by Prem Kumar-India

XI.28. If ( ) ( ) ( ) then find:

Proposed by Marian Ursărescu – Romania

XI.29 For (.∏

/ .∑ .

/

/*

, find:

:

:

.

/;

;

Proposed by Marian Ursărescu – Romania

XI.30. ( ) . Find: ( ) (( ) )

Proposed by Marian Ursărescu – Romania

XI.31. If ( ) then find: ( )

Proposed by Marian Ursărescu – Romania

XI.32. ( ) . Find: ( )

Proposed by Marian Ursărescu – Romania

XI.33. ( ) . Find: ( )

Proposed by Marian Ursărescu – Romania

XI.34. If ( ) then: ( ) ( )

Proposed by Marian Ursărescu – Romania

XI.35 ( ) ( ) . Find: ( )

Proposed by Marian Ursărescu – Romania

XI.36. Find all continuous functions such that:

( ) .

√ / – fixed

Proposed by Marian Ursărescu – Romania

XI.37. If ( ) ( ) then:

Proposed by Marian Ursărescu – Romania

Page 76: R. M. M. - 27...Tuzson, Florin Popovici, Viorel Gh. Vodă, Ovidiu Pop, E. Grosswald (USA), Leroy F. Meyers (USA), Francisco Bellot (Spain). Mihály encze contributed to ’’Gamma’’with

Romanian Mathematical Society-Mehedinți Branch 2020

75 ROMANIAN MATHEMATICAL MAGAZINE NR. 27

XI.38. If ( ) then:

( ( ) (

* *

Proposed by Marian Ursărescu – Romania

XI.39. ( ) ( ) ( ) . Find: ( )

Proposed by Marian Ursărescu – Romania

XI.40. Prove that in any triangle, the following inequality holds:

(

* ( )

with the usual notations in triangle.

Proposed by Radu Diaconu – Romania

XI.41. Prove that in any triangle, the following inequality holds:

∑(

*

* +

with the usual notations in triangle.

Proposed by Radu Diaconu, Emil Popa – Romania

XI.42. Prove that in any triangle, the following inequality holds:

(

* ( )

with the usual notations in triangle.

Proposed by Radu Diaconu – Romania

XI.43. In the following relationship holds:

( ) .

/ .

/

Proposed by Daniel Sitaru,Cristian Moanță – Romania

XI.44. If then:

Page 77: R. M. M. - 27...Tuzson, Florin Popovici, Viorel Gh. Vodă, Ovidiu Pop, E. Grosswald (USA), Leroy F. Meyers (USA), Francisco Bellot (Spain). Mihály encze contributed to ’’Gamma’’with

Romanian Mathematical Society-Mehedinți Branch 2020

76 ROMANIAN MATHEMATICAL MAGAZINE NR. 27

:∑:√

√ ;

; ( )( )( )

Proposed by Daniel Sitaru,Dana Cotfasă – Romania

XI.45. If then:

( )

( )

( )

( )

Proposed by Daniel Sitaru,Eugenia Turcu – Romania

XI.46. If then:

√ √

√ √

Proposed by Daniel Sitaru – Romania XI.47. Find:

(. /

:

∑:

. /

.

/;

;,

Proposed by Daniel Sitaru – Romania

XI.48. If in

( ) ( ) ( )

then:

Proposed by Daniel Sitaru – Romania XI.49.

(

:

;

(

+

)

((

+:

;,

Find:

(∏

+

Proposed by Daniel Sitaru – Romania

XI.50. {

⁄ * + * +

( ) }

Page 78: R. M. M. - 27...Tuzson, Florin Popovici, Viorel Gh. Vodă, Ovidiu Pop, E. Grosswald (USA), Leroy F. Meyers (USA), Francisco Bellot (Spain). Mihály encze contributed to ’’Gamma’’with

Romanian Mathematical Society-Mehedinți Branch 2020

77 ROMANIAN MATHEMATICAL MAGAZINE NR. 27

Find: .

/

Proposed by Daniel Sitaru – Romania

XI.51. Find:

:

.( )

( ) ( )

/

( ) ( ) ( );

Proposed by Daniel Sitaru – Romania

XI.52. If then:

. ( )

( )/. ( )

( )/ . ( )

( )/

Proposed by Daniel Sitaru – Romania

XI.53. Solve for real numbers:

( )

Proposed by Rovsen Pirguliyev-Azerbaijan

XI.54. Find such that: √( )( ) (√ √ )

Proposed by Rovsen Pirguliyev-Azerbaijan

XI.55. If

* + then:

√∏

Proposed by Rovsen Pirguliyev-Azerbaijan

XI.56. Prove without softs:

∫(

( )

( ) *

Proposed by Rovsen Pirguliyev-Azerbaijan

XI.57. If

then:

Page 79: R. M. M. - 27...Tuzson, Florin Popovici, Viorel Gh. Vodă, Ovidiu Pop, E. Grosswald (USA), Leroy F. Meyers (USA), Francisco Bellot (Spain). Mihály encze contributed to ’’Gamma’’with

Romanian Mathematical Society-Mehedinți Branch 2020

78 ROMANIAN MATHEMATICAL MAGAZINE NR. 27

Proposed by Seyran Ibrahimov-Azerbaijan

All solutions for proposed problems can be finded on the http//:www.ssmrmh.ro which is the adress of Romanian Mathematical

Magazine-Interactive Journal.

12-CLASS-STANDARD

XII.1. If and , find:

( √ √ √ √

,

Proposed by Florică Anastase – Romania XII.2. Prove:

∫( )

√( √ ) ( √ )

Proposed by Florică Anastase – Romania

XII.3. For , prove that ( ) such that:

√( )

( )

Proposed by Florică Anastase – Romania XII.4. Prove that:

Proposed by Florică Anastase – Romania XII.5. Prove that:

Page 80: R. M. M. - 27...Tuzson, Florin Popovici, Viorel Gh. Vodă, Ovidiu Pop, E. Grosswald (USA), Leroy F. Meyers (USA), Francisco Bellot (Spain). Mihály encze contributed to ’’Gamma’’with

Romanian Mathematical Society-Mehedinți Branch 2020

79 ROMANIAN MATHEMATICAL MAGAZINE NR. 27

.

/

Proposed by Florică Anastase – Romania

XII.6. ( ( ))( ( )) ( ) . Find:

( ) ∫4( ) ( ) ( ) ( )

( ( ))( ( ))5

Proposed by Daniel Sitaru – Romania

XII.7. Find:

(

:∑√

∑(√ √ )

;,

Proposed by Daniel Sitaru – Romania XII.8. Find:

(

( )∑4 4

55

+

Proposed by Daniel Sitaru – Romania

XII.9. If

then:

∫4 ( ) ( )

5

( )

Proposed by Daniel Sitaru,Ionuț Ivănescu – Romania

XII.10. If √

then:

∫∫:

.

/;

( )( )

.

/

Proposed by Daniel Sitaru – Romania

XII.11. If are all distinct such that and

( ) ∑ ∑

∑ ∑ ( )

where then prove that:

( ) (∏( )

+

Proposed by Daniel Sitaru – Romania XII.12. If then:

Page 81: R. M. M. - 27...Tuzson, Florin Popovici, Viorel Gh. Vodă, Ovidiu Pop, E. Grosswald (USA), Leroy F. Meyers (USA), Francisco Bellot (Spain). Mihály encze contributed to ’’Gamma’’with

Romanian Mathematical Society-Mehedinți Branch 2020

80 ROMANIAN MATHEMATICAL MAGAZINE NR. 27

Proposed by Daniel Sitaru – Romania XII.13. If then:

(

)

(

)

( ∫

,

Proposed by Daniel Sitaru – Romania XII.14. Let * + be the sequence of all positive integers which do not contain the digits

zero. Prove that ⌈

⌉ 0

1, where ( ).

Note: ∑ . 0

1 /

, -

Proposed by Prem Kumar-India

XII.15. Integrate: ∫ ( √ )

Proposed by Prem Kumar-India

XII.16. If , - ( ) , continuous then:

∫:∑(( ( ) ( ))

( ) ( ) ( ) ( )+

;

( )

Proposed by Daniel Sitaru – Romania

XII.17. Find:

∑ (∑( (

* (

**

+

Proposed by Daniel Sitaru – Romania

XII.18. Find:

:

( )( )( )

;

Proposed by Daniel Sitaru – Romania

XII.19. , - , continuous, ∫ ( )

. Find:

∑ :

:

∑ (

* (

* (

*

;;

Proposed by Daniel Sitaru – Romania

Page 82: R. M. M. - 27...Tuzson, Florin Popovici, Viorel Gh. Vodă, Ovidiu Pop, E. Grosswald (USA), Leroy F. Meyers (USA), Francisco Bellot (Spain). Mihály encze contributed to ’’Gamma’’with

Romanian Mathematical Society-Mehedinți Branch 2020

81 ROMANIAN MATHEMATICAL MAGAZINE NR. 27

XII.20. Solve the following system of congruences.

{

( )

( )

( )

( )

Proposed by Naren Bhandari-Nepal XII.21. Find the number of ordered quadruples of positive integers ( ) such that the following holds:

, and are primes.

Proposed by Naren Bhandari-Nepal XII.22. Find:

∫ √

Proposed by Radu Diaconu – Romania XII.23. If then:

:∏:∫. √ /

;

;

∏(

*

Proposed by Daniel Sitaru – Romania XII.24. Find:

(:∫(

*

;:∫(

*

;:∫(

*

;,

Proposed by Daniel Sitaru – Romania XII.25. Find:

(

∫ 4

5

√ ∑ .

/

∫ 4

5

√ ∑ .

/

)

Proposed by Daniel Sitaru – Romania

XII.26. Find:

Page 83: R. M. M. - 27...Tuzson, Florin Popovici, Viorel Gh. Vodă, Ovidiu Pop, E. Grosswald (USA), Leroy F. Meyers (USA), Francisco Bellot (Spain). Mihály encze contributed to ’’Gamma’’with

Romanian Mathematical Society-Mehedinți Branch 2020

82 ROMANIAN MATHEMATICAL MAGAZINE NR. 27

:∑∑∑∑

( )( )( )( )

;

Proposed by Daniel Sitaru – Romania

XII.27. Find:

:∫(

*

;

Proposed by Daniel Sitaru – Romania

XII.28. If

then:

( ) ( ):∫

;

∫ .

/

:∫

;:∫

;

Proposed by Daniel Sitaru – Romania

XII.29.

( ) ∫4

5

( ) ∫4

5

Prove that:

( ) ( )

0

1

Proposed by Daniel Sitaru – Romania

XII.30. If

then: ∫

( )

Find: ∫ (( ) )

Proposed by Seyran Ibrahimov-Azerbaijan

XII.31. Find (( )) .

/ such that:

( ) ( )

Proposed by Seyran Ibrahimov-Azerbaijan

XII.32. Find:

Page 84: R. M. M. - 27...Tuzson, Florin Popovici, Viorel Gh. Vodă, Ovidiu Pop, E. Grosswald (USA), Leroy F. Meyers (USA), Francisco Bellot (Spain). Mihály encze contributed to ’’Gamma’’with

Romanian Mathematical Society-Mehedinți Branch 2020

83 ROMANIAN MATHEMATICAL MAGAZINE NR. 27

∫ .0

( ) ( )

1/

, - great integer function.

Proposed by Jalil Hajimir-Canada

XII.33. Prove:

∫√ .

/

Proposed by Jalil Hajimir-Canada

All solutions for proposed problems can be finded on the http//:www.ssmrmh.ro which is the adress of Romanian Mathematical

Magazine-Interactive Journal.

UNDERGRADUATE PROBLEMS

U.1. Find:

:∫ ( )

;

Proposed by Ajao Yinka-Nigeria U.2. Find:

∫ ( )

Proposed by Ajao Yinka-Nigeria U.3. Find:

Proposed by Ajao Yinka-Nigeria U.4. Find:

:∫ ( )

;

Proposed by Ajao Yinka-Nigeria

Page 85: R. M. M. - 27...Tuzson, Florin Popovici, Viorel Gh. Vodă, Ovidiu Pop, E. Grosswald (USA), Leroy F. Meyers (USA), Francisco Bellot (Spain). Mihály encze contributed to ’’Gamma’’with

Romanian Mathematical Society-Mehedinți Branch 2020

84 ROMANIAN MATHEMATICAL MAGAZINE NR. 27

U.5. Find:

( )

Proposed by Ajao Yinka-Nigeria U.6. ( ) . Find:

∑( )

Proposed by Daniel Sitaru – Romania U.7. , - ( ) – continuous. Find ⏟

such that:

∫∫∫(( ( ) ( )) ( )

( ( ) ( )) ( )

( ( ) ( )) ( )

∫ ( )

Proposed by Daniel Sitaru,Claudia Nănuți – Romania

U.8. If then:

∫∫ √ ( )

( ) √ ( )

Proposed by Daniel Sitaru,Marian Voinea – Romania

U.9. If then:

∫∫∫4

√ 5

(

*

( )

Proposed by Daniel Sitaru,Marian Voinea – Romania

U.10. If then:

( )( )∫∫4

5

Proposed by Daniel Sitaru,Claudia Nănuți – Romania

U.11. Evaluate the following sum:

( )

Proposed by Prem Kumar-India

U.12. Prove that . ∑

( ) /

In fact, its value is Niven’s constant that is equal to Proposed by Prem Kumar-India

U.13. Prove that:

Page 86: R. M. M. - 27...Tuzson, Florin Popovici, Viorel Gh. Vodă, Ovidiu Pop, E. Grosswald (USA), Leroy F. Meyers (USA), Francisco Bellot (Spain). Mihály encze contributed to ’’Gamma’’with

Romanian Mathematical Society-Mehedinți Branch 2020

85 ROMANIAN MATHEMATICAL MAGAZINE NR. 27

( )∏

√ ( )

The product on the left-hand side extends over all prime numbers . Proposed by Prem Kumar-India

U.14. Prove that

Proposed by Prem Kumar-India U.15. Prove that:

( ) ( ∑

( )

+

Proposed by Prem Kumar-India

U.16. Find all values of for which .

/ is not divisible by .

Proposed by Prem Kumar-India U.17. Evaluate the following sum:

( )

Proposed by Prem Kumar-India

U.18. For any real number and a positive integer prove that:

| ( )

( )

( )

( )

|

Proposed by K. Srinivasa Raghava-AIRMC-India

U.19. Prove the following:

∫ 4

( ) 5

( )

∫ 4

( ) 5

( )

( ( ( )) )

– Catalan constant Proposed by K. Srinivasa Raghava-AIRMC-India

U.20. Evaluate the integral in a closed – form:

∫ ( )

( )

Proposed by K. Srinivasa Raghava-AIRMC-India U.21. Prove the relation:

∫ .√ √ /

( ) ( )

Page 87: R. M. M. - 27...Tuzson, Florin Popovici, Viorel Gh. Vodă, Ovidiu Pop, E. Grosswald (USA), Leroy F. Meyers (USA), Francisco Bellot (Spain). Mihály encze contributed to ’’Gamma’’with

Romanian Mathematical Society-Mehedinți Branch 2020

86 ROMANIAN MATHEMATICAL MAGAZINE NR. 27

( ) ( ) ( )

.

/ ( ) ( )

where ( ) ( ) ∫ ( )

Proposed by K. Srinivasa Raghava-AIRMC-India

U.22. Compute the integral in a closed – form:

∫ 4√

5

Proposed by K. Srinivasa Raghava-AIRMC-India U.23. If

( ) ∫ ( )

( )

then, ∑ , ( )-( )

, ( )-( )

, ( )-( ) – Laplace transform of ( ) with variable , ( )-( ) – Mellin transform of ( ) with variable

Proposed by K. Srinivasa Raghava-AIRMC-India U.24. Let – Harmonic Number

( ) ∑ ∑( )

( )( )( )

then find the shortest solutions to the following values ( ) ( )

( ) ( )

( ) ( )

( ) ( )

Proposed by K. Srinivasa Raghava-AIRMC-India

U.25. For any complex number ( )

( ) .

/

( )

( )

( )

( )

Proposed by K. Srinivasa Raghava-AIRMC-India

U.26. Prove the following relation

∫ (

*

( ( )

* ( )

4 4 √ √ 5 5

Proposed by K. Srinivasa Raghava-AIRMC-India

U.27. Prove the integral relation and evaluate in a closed form:

Page 88: R. M. M. - 27...Tuzson, Florin Popovici, Viorel Gh. Vodă, Ovidiu Pop, E. Grosswald (USA), Leroy F. Meyers (USA), Francisco Bellot (Spain). Mihály encze contributed to ’’Gamma’’with

Romanian Mathematical Society-Mehedinți Branch 2020

87 ROMANIAN MATHEMATICAL MAGAZINE NR. 27

∫∫ .

/

4 ( )

( ) 5 ∫

( ( ))

Proposed by K. Srinivasa Raghava-AIRMC-India

U.28. Find the Principal value of the integral:

∫ ( )

.

/

( )

.

/ ( )

Proposed by K. Srinivasa Raghava-AIRMC-India

U.29. Let be the angles of a triangle then show that:

.

/ .

/ .

/

.

/ .

/ .

/ .

/

Proposed by K. Srinivasa Raghava-AIRMC-India

U.30. Let be real numbers and if: ( ) ( ) ( )( )

and ( ) ( ) ( )( ) then find the value of the expression

4

5

(

*

Proposed by K. Srinivasa Raghava-AIRMC-India U.31. Prove the following relation:

∫ ( )√ ( )

(√ ( ))

√√ √

4 4√ √

5 (

√ *5

Proposed by K. Srinivasa Raghava-AIRMC-India

U.32. Evaluate the integral in a closed form:

:∫ ( )

( )

( ) ;

Proposed by K. Srinivasa Raghava-AIRMC-India

U.33. Find the shortest way to prove that:

∫ (√ )

( ) ( )

( )

where is the Catalan Constant Proposed by K. Srinivasa Raghava-AIRMC-India

Page 89: R. M. M. - 27...Tuzson, Florin Popovici, Viorel Gh. Vodă, Ovidiu Pop, E. Grosswald (USA), Leroy F. Meyers (USA), Francisco Bellot (Spain). Mihály encze contributed to ’’Gamma’’with

Romanian Mathematical Society-Mehedinți Branch 2020

88 ROMANIAN MATHEMATICAL MAGAZINE NR. 27

U.34. If we define the function:

( ) ∫ ( )

( )

( )

then evaluate the integral in a closed – form:

∫ :∫ ( )

;

Proposed by K. Srinivasa Raghava-AIRMC-India

U.35. Find:

:∫ 4 ( )

5

;

Proposed by Mohamed Arahman Jama-Somalia

U.36. Find:

( )

. .

//

Proposed by Mohamed Arahman Jama-Somalia

U.37. Let be the solution of the following equation: ( )

.

/ ( ) .

Find:

:

( )∫

( )

;

Proposed by Mohamed Arahman Jama-Somalia

U.38. Find:

:∫( )( )

;

Proposed by Mohamed Arahman Jama-Somalia

U.39. Find:

∫(∑(

( ) *

+

Proposed by Mohamed Arahman Jama-Somalia

Page 90: R. M. M. - 27...Tuzson, Florin Popovici, Viorel Gh. Vodă, Ovidiu Pop, E. Grosswald (USA), Leroy F. Meyers (USA), Francisco Bellot (Spain). Mihály encze contributed to ’’Gamma’’with

Romanian Mathematical Society-Mehedinți Branch 2020

89 ROMANIAN MATHEMATICAL MAGAZINE NR. 27

U.40. Find:

∑(

( ( )

* *

Proposed by Mohamed Arahman Jama-Somalia

U.41. Find:

∑4( ) ( )

( ) 5

Proposed by Mohamed Arahman Jama-Somalia

U.42. Find:

∫ :∑ :

(∑

(

*+;

;

Proposed by Mohamed Arahman Jama-Somalia

U.43. For ( ) * +. Prove that:

∫. ( √ )

( ) /

Proposed by Mohammed Bouras – Maroc

U.44. Prove that:

∫ .( )

( )

/

Proposed by Mohammed Bouras – Maroc

U.45.

∫√ √

√ √ √ √ √ √ √ √

– Fibonacci number;

Prove that

is faster to give golden number then

Proposed by Mohammed Bouras – Maroc

Page 91: R. M. M. - 27...Tuzson, Florin Popovici, Viorel Gh. Vodă, Ovidiu Pop, E. Grosswald (USA), Leroy F. Meyers (USA), Francisco Bellot (Spain). Mihály encze contributed to ’’Gamma’’with

Romanian Mathematical Society-Mehedinți Branch 2020

90 ROMANIAN MATHEMATICAL MAGAZINE NR. 27

U.46. If

then:

( ) √ ∫∫∫

4

( )5

Proposed by Daniel Sitaru – Romania

U.47. Evaluate:

∫ ( )

Proposed by Abdul Mukhtar-Nigeria U.48.

∫√ ( ) ( )

√ ( ) ( ) ( )

Proposed by Abdul Mukhtar-Nigeria U.49. Prove:

∫ (

√ *

4 ( )

5 (

*

Proposed by Abdul Mukhtar-Nigeria U.50.

∫∑

( )

( ) ( )

Proposed by Abdul Mukhtar-Nigeria

U.51. Find:

:∫ ( ( ) )

;

Proposed by Abdul Mukhtar-Nigeria U.52. Evaluate:

Page 92: R. M. M. - 27...Tuzson, Florin Popovici, Viorel Gh. Vodă, Ovidiu Pop, E. Grosswald (USA), Leroy F. Meyers (USA), Francisco Bellot (Spain). Mihály encze contributed to ’’Gamma’’with

Romanian Mathematical Society-Mehedinți Branch 2020

91 ROMANIAN MATHEMATICAL MAGAZINE NR. 27

∫ ( (

**

Proposed by Abdul Mukhtar-Nigeria U.53.

∫ ( ) ( ( ) ( ))

Proposed by Abdul Mukhtar-Nigeria U.54.

∫ ( )

∫ ( )

∫ ( )

( )

( )

where ( ) is the Riemann zeta function.

Proposed by Abdul Hafeez Ayinde-Nigeria

U.55.

∫( ( ) ( ))

Proposed by Abdul Hafeez Ayinde-Nigeria

U.56. Find:

∫4

√ 5

Proposed by Abdul Hafeez Ayinde-Nigeria

U.57. ( ) ∫ .

/

( ). Find:

∫4 ( )

√ 5

Proposed by Abdul Hafeez Ayinde-Nigeria

U.58. Find:

Page 93: R. M. M. - 27...Tuzson, Florin Popovici, Viorel Gh. Vodă, Ovidiu Pop, E. Grosswald (USA), Leroy F. Meyers (USA), Francisco Bellot (Spain). Mihály encze contributed to ’’Gamma’’with

Romanian Mathematical Society-Mehedinți Branch 2020

92 ROMANIAN MATHEMATICAL MAGAZINE NR. 27

( ) ∫ (

( )( )*

Proposed by Abdul Hafeez Ayinde-Nigeria

U.59. Prove without the use of Harmonic series that:

∫ ( )

( )

Proposed by Abdul Hafeez Ayinde-Nigeria

U.60. Prove that:

( )

( )

( )

( )

( )

( )

( )

where is catalan’s constant.

Proposed by Abdul Hafeez Ayinde-Nigeria

U.61. A mixture of integrals and summations:

∫ ( )

∫ ( )

∫ ( )

∫ ( )

( )

( )

∫ ( )

( )

( )

( ) ( )

( )

Where ( ) is the Riemann zeta function, is Catalan’s constant, is the Harmonic Number.

Proposed by Abdul Hafeez Ayinde-Nigeria

U.62.

Page 94: R. M. M. - 27...Tuzson, Florin Popovici, Viorel Gh. Vodă, Ovidiu Pop, E. Grosswald (USA), Leroy F. Meyers (USA), Francisco Bellot (Spain). Mihály encze contributed to ’’Gamma’’with

Romanian Mathematical Society-Mehedinți Branch 2020

93 ROMANIAN MATHEMATICAL MAGAZINE NR. 27

∫ . √ /

Find .

/ and prove that:

(( ) ) (

* ( ) ( )

Proposed by Abdul Hafeez Ayinde-Nigeria

U.63. Inspired by Prof. Daniel Sitaru

Prove that:

∑(∑ (

( )( ) *

+

( )√

Proposed by Naren Bhandari-Nepal

U.64. Define for all ( ) √ √ √ √ ( )

then prove that:

∫ ( )

( )

√ √ √ √ √

√ √

Proposed by Naren Bhandari-Nepal

U.65. For all let:

( )

Prove that:

∑ ( ) ( ) ( )

( ) ( )

( )

( )

Proposed by Naren Bhandari-Nepal

Page 95: R. M. M. - 27...Tuzson, Florin Popovici, Viorel Gh. Vodă, Ovidiu Pop, E. Grosswald (USA), Leroy F. Meyers (USA), Francisco Bellot (Spain). Mihály encze contributed to ’’Gamma’’with

Romanian Mathematical Society-Mehedinți Branch 2020

94 ROMANIAN MATHEMATICAL MAGAZINE NR. 27

U.66.

( ) ∑ . .

//

.

/

Prove that:

4

5

( ) ( ) ( )

Proposed by Daniel Sitaru – Romania U.67. Find:

∑(4

( )

5

( ) +

Proposed by Daniel Sitaru – Romania

U.68. If then:

∫∫ (

*

( )∫

∫∫ (

*

( )∫

Proposed by Daniel Sitaru – Romania U.69. If , different in pairs then:

( ) ( ) ( ) :∏:√

;

;

( ) ∑(

4

5

+

Proposed by Daniel Sitaru – Romania

U.70.

∫ ( )

( )

Proposed by Abdul Mukhtar-Nigeria

Page 96: R. M. M. - 27...Tuzson, Florin Popovici, Viorel Gh. Vodă, Ovidiu Pop, E. Grosswald (USA), Leroy F. Meyers (USA), Francisco Bellot (Spain). Mihály encze contributed to ’’Gamma’’with

Romanian Mathematical Society-Mehedinți Branch 2020

95 ROMANIAN MATHEMATICAL MAGAZINE NR. 27

U.71. Let ( ) ( ) and ( ) ∫ ( )

prove that ∑ ( )

.

√ /

Proposed by Abdul Mukhtar-Nigeria

U.72.

∫ ( )

( ( ) ( ))

Proposed by Abdul Mukhtar-Nigeria

U.73. Find:

:∫ 4

5

;

Proposed by Abdul Mukhtar-Nigeria

U.74.

∫ ( ) ( ( ) ( ))

Proposed by Abdul Mukhtar-Nigeria

U.75. Find:

∫ (

√ *

Proposed by Abdul Mukhtar-Nigeria

U.76. Prove that:

∫ ( )

(

*

Proposed by Abdul Mukhtar-Nigeria

U.77. Prove:

∫∫ ∫

6, -

, - 7

Proposed by Jalil Hajimir-Canada

Page 97: R. M. M. - 27...Tuzson, Florin Popovici, Viorel Gh. Vodă, Ovidiu Pop, E. Grosswald (USA), Leroy F. Meyers (USA), Francisco Bellot (Spain). Mihály encze contributed to ’’Gamma’’with

Romanian Mathematical Society-Mehedinți Branch 2020

96 ROMANIAN MATHEMATICAL MAGAZINE NR. 27

U.78. Prove:

∫∫∫.

/

Proposed by Jalil Hajimir-Canada

U.79. Find:

( ) ∫ ∫ ( , -)

, - - great integer function.

Proposed by Jalil Hajimir-Canada

U.80. Find the points on the surface that are closest to the origin.

Proposed by Jalil Hajimir-Canada

U.81. Prove without softs:

∫∫∫4 ( ) ( ) ( )

5

Proposed by Jalil Hajimir-Canada

U.82.

∫∫ , -

, -

, -

, - is the greatest integer part of

Proposed by Jalil Hajimir-Canada

U.83. Find:

∫4 ( ) ( )

5

Proposed by Cornel Ioan Vălean – Romania

U.84. Find:

∫4 ( )

5

Proposed by Cornel Ioan Vălean – Romania

Page 98: R. M. M. - 27...Tuzson, Florin Popovici, Viorel Gh. Vodă, Ovidiu Pop, E. Grosswald (USA), Leroy F. Meyers (USA), Francisco Bellot (Spain). Mihály encze contributed to ’’Gamma’’with

Romanian Mathematical Society-Mehedinți Branch 2020

97 ROMANIAN MATHEMATICAL MAGAZINE NR. 27

All solutions for proposed problems can be finded on the http//:www.ssmrmh.ro which is the adress of Romanian Mathematical

Magazine-Interactive Journal.

ROMANIAN MATHEMATICAL MAGAZINE-R.M.M.-Winter 2020

PROBLEMS FOR JUNIORS

JP.271. If then:

( ) ( ) ( )

Proposed by Marin Chirciu – Romania

JP.272. If then:

√ √ √ √

Proposed by Hung Nguyen Viet – Vietnam

JP.273. If then:

( )

Proposed by Hung Nguyen Viet – Vietnam

JP.274. If then:

( )( ) ( )

Proposed by Marin Chirciu – Romania

JP.275. If in then:

Proposed by Daniel Sitaru – Romania

JP.276. In the following relationship holds:

Proposed by Marin Chirciu – Romania

Page 99: R. M. M. - 27...Tuzson, Florin Popovici, Viorel Gh. Vodă, Ovidiu Pop, E. Grosswald (USA), Leroy F. Meyers (USA), Francisco Bellot (Spain). Mihály encze contributed to ’’Gamma’’with

Romanian Mathematical Society-Mehedinți Branch 2020

98 ROMANIAN MATHEMATICAL MAGAZINE NR. 27

JP.277. In the following relationship holds:

(

*

(

*

(

*

Proposed by Marin Chirciu – Romania

JP.278. Solve for real numbers ( ; fixex):

√ ( )

Proposed by Marin Chirciu – Romania

JP.279. In the following relationship holds:

( )

( )

( )

Proposed by Hung Nguyen Viet – Vietnam

JP.280. In the following relationship holds:

√ √

( )

Proposed by Hung Nguyen Viet – Vietnam

JP.281. If then:

( )

( )

( )

√ √

Proposed by Hung Nguyen Viet – Vietnam

JP.282. If then:

( ) ( )

Proposed by Daniel Sitaru – Romania

JP.283. If then:

∑ ( )

Proposed by Daniel Sitaru – Romania

JP.284. In acute the following relationship holds:

√ √ √

√ √ √ √ .

/

Proposed by Marian Ursărescu – Romania

JP.285. In the following relationship holds:

Page 100: R. M. M. - 27...Tuzson, Florin Popovici, Viorel Gh. Vodă, Ovidiu Pop, E. Grosswald (USA), Leroy F. Meyers (USA), Francisco Bellot (Spain). Mihály encze contributed to ’’Gamma’’with

Romanian Mathematical Society-Mehedinți Branch 2020

99 ROMANIAN MATHEMATICAL MAGAZINE NR. 27

Proposed by Marian Ursărescu – Romania

PROBLEMS FOR SENIORS

SP.271. If

then:

Proposed by Marin Chirciu – Romania

SP.272. In the following relationship holds:

4

5

Proposed by George Apostolopoulos – Greece

SP.273. If then:

When does the equality holds?

Proposed by Daniel Sitaru – Romania

SP.274. If in

then the following relationship holds:

Proposed by Daniel Sitaru – Romania

SP.275. In the following relationship holds:

(

*

(

*

(

*

Proposed by Hung Nguyen Viet – Vietnam

SP.276. If then:

∑( )( )

( )

Proposed by Marin Chirciu – Romania

SP.277. In the following relationship holds:

Page 101: R. M. M. - 27...Tuzson, Florin Popovici, Viorel Gh. Vodă, Ovidiu Pop, E. Grosswald (USA), Leroy F. Meyers (USA), Francisco Bellot (Spain). Mihály encze contributed to ’’Gamma’’with

Romanian Mathematical Society-Mehedinți Branch 2020

100 ROMANIAN MATHEMATICAL MAGAZINE NR. 27

(

*

∑:√

;

Proposed by George Apostolopoulos – Greece

SP.278. Let be 0

1 ( )

. Find Imf.

Proposed by Marin Chirciu – Romania

SP.279. If in – Brocard angle then the following relationship holds:

Proposed by Vasile Jiglău – Romania

SP.280. If * + * + * +

then:

, - * + , - * + , - * + (, - , - , - )

* + , - , - great integer function

Proposed by Daniel Sitaru – Romania

SP.281. If .

/ then:

(

4√

5

:√

;

)

(

4√

5

:√

;

)

:√

;

Proposed by Daniel Sitaru – Romania

SP.282. If in – orthocenter; bisectors of angles respectively

( ) ( ) ( ) then the following relationship holds:

, -

, - .

/

Proposed by Marian Ursărescu – Romania

SP.283. Find such that:

Proposed by Daniel Sitaru – Romania

Page 102: R. M. M. - 27...Tuzson, Florin Popovici, Viorel Gh. Vodă, Ovidiu Pop, E. Grosswald (USA), Leroy F. Meyers (USA), Francisco Bellot (Spain). Mihály encze contributed to ’’Gamma’’with

Romanian Mathematical Society-Mehedinți Branch 2020

101 ROMANIAN MATHEMATICAL MAGAZINE NR. 27

SP.284. In the following relationship holds:

( )

( )

( )

Proposed by George Apostolopoulos – Greece

SP.285. In the following relationship holds:

Proposed by George Apostolopoulos – Greece

UNDERGRADUATE PROBLEMS

UP.271. If

then:

∫∫∫. .

/ .

/ .

//

( ) ( )

Proposed by Daniel Sitaru – Romania UP.272. Prove without softs:

∫∫∫( (√ ))

Proposed by Florentin Vișescu – Romania

UP.273. In acute the following relationship hold:

(√ ) (√ ) (√ )

Proposed by Florentin Vișescu – Romania

UP.274. If

. /

. /

( )

. / then find:

4 √

5

Proposed by Florică Anastase – Romania UP.275. Find:

:

∑∑∑∑( )

;

Proposed by Daniel Sitaru – Romania UP.276. Find:

:∑∑∑∑(

*

;

Proposed by Daniel Sitaru – Romania

Page 103: R. M. M. - 27...Tuzson, Florin Popovici, Viorel Gh. Vodă, Ovidiu Pop, E. Grosswald (USA), Leroy F. Meyers (USA), Francisco Bellot (Spain). Mihály encze contributed to ’’Gamma’’with

Romanian Mathematical Society-Mehedinți Branch 2020

102 ROMANIAN MATHEMATICAL MAGAZINE NR. 27

UP.277. Find:

(∑ (

√( ) *

,

Proposed by Marian Ursărescu – Romania

UP.278. If then:

∫∫( ( ))

( )

Proposed by Daniel Sitaru – Romania

UP.279. If ( ) ( ) . ( )

( )/ and exists

(( ( ))

* then:

:(( ( ))

( ( ))

* ( ( ))

;

Proposed by D.M. Bătinețu – Giurgiu, Neculai Stanciu – Romania

UP.280. Let be sides in ( ) ( ) ( ) sequence of positive numbers such that:

( )

(

*

(

*

Prove that:

4 √

√ √

5

Proposed by D.M. Bătinețu – Giurgiu, Neculai Stanciu – Romania

UP.281. If ( ) ( ) .

/ √

√ √ √ √

√ then find:

4( )

5

Proposed by D.M. Bătinețu – Giurgiu, Daniel Sitaru – Romania

UP.282. If ( ) ( ) ( ) ( )

(

*

(

*

.

/ then:

Page 104: R. M. M. - 27...Tuzson, Florin Popovici, Viorel Gh. Vodă, Ovidiu Pop, E. Grosswald (USA), Leroy F. Meyers (USA), Francisco Bellot (Spain). Mihály encze contributed to ’’Gamma’’with

Romanian Mathematical Society-Mehedinți Branch 2020

103 ROMANIAN MATHEMATICAL MAGAZINE NR. 27

(

( )

)

( )

Proposed by D.M. Bătinețu – Giurgiu, Neculai Stanciu – Romania

UP.283. In the following relationship holds:

( )

( )

( )

Proposed by George Apostolopoulos – Greece

UP.284. In the following relationship holds:

( ) ∑

( )( )

Proposed by George Apostolopoulos – Greece

UP.285. In the following relationship holds:

4

5

Proposed by George Apostolopoulos – Greece

All solutions for proposed problems can be finded on the http//:www.ssmrmh.ro which is the adress of Romanian Mathematical

Magazine-Interactive Journal.

Page 105: R. M. M. - 27...Tuzson, Florin Popovici, Viorel Gh. Vodă, Ovidiu Pop, E. Grosswald (USA), Leroy F. Meyers (USA), Francisco Bellot (Spain). Mihály encze contributed to ’’Gamma’’with

Romanian Mathematical Society-Mehedinți Branch 2020

104 ROMANIAN MATHEMATICAL MAGAZINE NR. 27

INDEX OF AUTHORS RMM-27

Nr.crt. Numele și prenumele Nr.crt. Numele și prenumele

1 DANIEL SITARU-ROMANIA 44 AJAO YINKA-NIGERIA

2 D.M.BĂTINEȚU-GIURGIU-ROMANIA 45 PREM KUMAR-INDIA

3 CLAUDIA NĂNUȚI-ROMANIA 46 MEHMET ȘAHIN-TURKEY

4 NECULAI STANCIU-ROMANIA 47 SEYRAN IBRAHIMOV-AZERBAIJAN

5 MARIAN URSĂRESCU-ROMANIA 48 NGUYEN VIET HUNG-VIETNAM

6 BOGDAN FUȘTEI-ROMANIA 49 SRINIVASA RAGHAVA-INDIA

7 FLORICĂ ANASTASE-ROMANIA 50 NAREN BHANDARI-NEPAL

8 MARIN CHIRCIU-ROMANIA 51 MOHAMMED BOURAS-MAROC

9 VASILE BURUIANĂ-ROMANIA 52 MOHAMED ARAHMAN JAMA-SOMALIA

10 MARIAN DINCĂ-ROMANIA 53 VASILE JIGLĂU-ROMANIA

11 FLORENTIN VIȘESCU-ROMANIA 54 JALIL HAJIMIR-CANADA

12 ȘTEFAN MARICA-ROMANIA 55 ROVSEN PIRGULIYEV-AZERBAIJAN

13 ENE CRISTINA-ROMANIA 56 GEORGE APOSTOLOPOULOS-GREECE

14 GHEORGHE CALAFETEANU-ROMANIA 57 ABDUL MUKHTAR-NIGERIA

15 ELENA NICU-ROMANIA 58 ABDUL HAFEEZ AYINDE-NIGERIA

16 PETRE STÂNGESCU-ROMANIA 29 MĂDĂLINA GIURGESCU-ROMANIA

17 IULIANA TRAȘCĂ-ROMANIA 60 ALINA GEORGIANA GHIȚĂ-ROMANIA

18 AUREL CHIRIȚĂ -ROMANIA 61 ALINA TIGAE-ROMANIA

19 CONSTANTIN IONICĂ-ROMANIA 62 AMELIA CURCĂ NĂSTĂSELU-ROMANIA

20 DANIEL STRETCU-ROMANIA 63 LAVINIA TRINCU-ROMANIA

21 MIREA MIHAELA MIOARA-ROMANIA 64 CAMELIA DANĂ-ROMANIA

22 GHEORGHE ALEXE-ROMANIA 65 MARIAN VOINEA-ROMANIA

23 GEORGE-FLORIN ȘERBAN-ROMANIA 66 CĂTĂLIN PANĂ-ROMANIA

24 LUCIAN TUȚESCU-ROMANIA 67 CLAUDIU CIULCU-ROMANIA

25 RADU DIACONU-ROMANIA 68 CRISTINA SPIRIDON-ROMANIA

26 EMIL POPA-ROMANIA 69 CONSTANTINA PRUNARU-ROMANIA

27 ALECU ORLANDO-ROMANIA 70 CORINA IONESCU-ROMANIA

28 CRISTIAN MOANȚĂ-ROMANIA 71 IULIA SANDA-ROMANIA

29 DAN GRIGORIE-ROMANIA 72 LAURA ZAHARIA-ROMANIA

30 DANA COTFASĂ-ROMANIA 73 LUIZA CREMENEANU-ROMANIA

31 DANIELA BELDEA-ROMANIA 74 MIHAELA DĂIANU-ROMANIA

32 CATALIN SPIRIDON-ROMANIA 75 MIHAELA NASCU-ROMANIA

33 DELIA POPESCU-ROMANIA 76 MIHAELA STĂNCELE-ROMANIA

34 DELIA SHNEIDER-ROMANIA 77 NICOLAE RADU-ROMANIA

35 DOINA CRISTINA CĂLINA-ROMANIA 78 NINETA OPRESCU-ROMANIA

36 DORINA GOICEANU-ROMANIA 79 TATIANA CRISTEA-ROMANIA

37 ELENA ALEXIE-ROMANIA 80 IOAN CORNEL VĂLEAN-ROMANIA

38 ELENA IACOB MEDA-ROMANIA 81 PATRICIA ANICUȚA BEȚIU-ROMANIA

39 EUGENIA TURCU-ROMANIA 82 ROXANA VASILE-ROMANIA

40 GABRIELA VASILE-ROMANIA 83 VIRGINIA GRIGORESCU-ROMANIA

41 ILEANA DUMA-ROMANIA 84 GHEORGHE BOROICA-ROMANIA

42 ILEANA STANCIU-ROMANIA 85 NICOLAE MUȘUROIA-ROMANIA

43 IONUȚ IVĂNESCU-ROMANIA 86 GHEORGHE STOICA-ROMANIA NOTĂ: Pentru a publica probleme propuse, articole și note matematice în RMM puteți trimite materialele pe mailul: [email protected]